You are on page 1of 79

Giáo viên chuyên Toán Star Education

No. 6 - 2021
Mục lục

1 Một số bài toán số học hay từ đề thi vào lớp 10 Chuyên Toán 3

2 Một số vấn đề cơ bản về Dãy số 15

3 Nhìn lại các bài toán hình phẳng trong đề tuyển sinh chuyên Toán PTNK 23

4 Các bài toán tổ hợp từ đề JBMO (tiếp theo) 39

5 Đề thi thử Toán không chuyên tuyển sinh 10 THPT 46

6 Đề thi thử Toán Chuyên tuyển sinh 10 THPT lần 1 53

7 Đề thi thử Toán Chuyên tuyển sinh 10 THPT lần 2 59

8 Đề kiểm tra cuối khóa lớp 10CĐ 65

9 Lời giải các câu VD-VDC đề minh họa THPT quốc gia 2021 72

1
2

NGUYỄN TĂNG VŨ - LÊ PHÚC LỮ - NGUYỄN CÔNG THÀNH

TẬP SAN TOÁN HỌC


STAR EDUCATION
Số thứ 06 - 2021

Vậy là chúng tôi đã đi được chặng đường 3 năm với tổng cộng 6 quyển tài liệu Tập
san Toán học STAR EDUCATION để gửi đến bạn đọc trên khắp cả nước. Trong
tập san lần này, nhóm tác giả dành các bài viết chuyên đề tập trung cho đối tượng
học sinh thi vào lớp 10 chuyên, bao gồm 3 chuyên đề song song với 3 đề thi thử (một
đề Toán không chuyên và hai đề Toán chuyên). Đồng thời, cũng sẽ có các nội dung
về Toán THPT Chuyên và đặc biệt là lời giải các câu VD-VDC trong đề Minh họa
cho kỳ thi THPT Quốc gia mà Bộ GD-ĐT đã công bố vào dịp cuối tháng 3 vừa rồi.

Bài viết trong tập san cũng được nhiều tác giả xây dựng với phong cách viết bài
khá phong phú, đa giảng; các lời giải trình bày chi tiết, chuẩn mực để các em học
sinh có thể tham khảo theo. Chỉ còn chưa đầy một tuần nữa là các em học sinh lớp
9 ở TPHCM sẽ bước vào kỳ thi tuyển sinh Chuyên Toán, cũng như các em học sinh
cùng lứa tuổi ở khắp cả nước. Các anh chị lớp 12 thì cũng đang ở giai đoạn ôn tập
quyết liệt để hướng đến kỳ thi tuyển sinh ĐH vào tháng 7 tới đây. Mong rằng quyển
tài liệu này sẽ góp một phần nào đó vào quá trình ôn luyện của các bạn.
Mọi đóng góp, bài viết xin gửi về các địa chỉ nguyentangvu@gmail.com hoặc
lephuclu@gmail.com.
Bản quyền thuộc trung tâm STAR EDUCATION, được đăng tải miễn phí trên
mạng. Mong rằng tài liệu này sẽ được đón nhận và được chia sẻ rộng rãi. Xin chân
thành cảm ơn.

Tập san Toán học STAR EDUCATION


Một số bài toán số học hay từ
đề thi vào lớp 10 Chuyên Toán

Nguyễn Tăng Vũ
(GV trường PTNK TPHCM)

Trong khi thì HSG TPHCM vừa qua có một điều đáng tiếc nhất là câu số học không có
trong đề thi, làm nhiều thí sinh khá hụt hẫng nhưng cũng làm nhiều thí sinh vui mừng,
vì số học luôn là câu hỏi hóc búa của mỗi kì thi. Có lẽ BTC cuộc thi muốn dành sự quan
tâm cho các câu hỏi thực tế nên phần số học bị bỏ qua.
Khác với kì thi HSG, kì thi tuyển sinh vào 10 thì đề thi luôn có đủ cả các phần: đại số, số
học, hình học và tổ hợp. Số học cũng như tổ hợp, luôn là phần khiến nhiều thí sinh gặp
khó khăn, trong bài viết nhỏ này, tôi xin giới thiệu lại một số bài toán số học đã được
cho trong các kì thi tuyển sinh của trường Phổ thông Năng khiếu, nơi tôi làm việc hơn
10 năm qua. Các bạn thí sinh chuẩn bị thi vào trường nên xem kĩ lời giải và cố gắng học
thật tốt phần này, điều đó sẽ giúp rất nhiều cơ hội trúng tuyển vào lớp chuyên toán.
Số học THCS thì nội dung quay xung quanh các phép chia hết, phương trình nghiệm
nguyên, số nguyên tố, số chính phương,... Việc đầu tiên là nắm chắc các tính chất của
phép chia hết, tính chất cơ bản nhất của số nguyên tố hay số chính phương. Bài toán chia
hết cũng xuất hiện nhiều lần trong đề thi, sau đây là một bài khá đơn giản nhưng hay.

1. Phân tích và bình luận các ví dụ

Ví dụ 1. (PTNK 2011 - Chuyên Toán) Cho các số nguyên a, b, c sao cho

2a + b, 2b + c, 2c + a

đều là các số chính phương (*).

1. Biết rằng có ít nhất một trong 3 số chính phương trên chia hết cho 3. Chứng
minh rằng (a − b)(b − c)(c − a) chia hết cho 27.

2. Tồn tại hay không các số a, b, c thỏa điều kiện (*) mà (a − b)(b − c)(c − a)
không chia hết cho 27?

Nhận xét. Đây là một bài toán chia hết, liên quan đến các số chính phương, để ý

3
4 NGUYỄN TĂNG VŨ

thấy chủ yếu là chia hết cho 3. Ta phải nghĩ đến một số chính phương chia 3 xảy ra
những trường hợp nào, từ đó thiết lập các tính chất đã biết:

ˆ Một số chính phương khi chia cho 3 dư 0 hoặc 1.

ˆ a2 + b2 chia hết cho 3 khi và chỉ khi a, b đồng thời chia hết cho 3.

ˆ Việc chứng minh tích chia hết cho 27, thì nghĩ đến việc ta cần chứng minh
a, b, c có cùng số dư khi chia cho 3, đó là trường hợp đơn giản nhất. Sau đây
là lời giải

Lời giải.

1. Giả sử 2a + b = m2 , 2b + c = n2 , 2c + a = p2 .
Cộng ba đẳng thức lại, ta được 3(a + b + c) = m2 + n2 + p2 . Suy ra m2 + n2 + p2
chia hết cho 3.
Ta thấy bình phương của một số nguyên khi chia cho 3 dư 1 hoặc 0. Do đó
nếu 1 trong 3 số, chẳng hạn m chia hết cho 3 thì n2 + p2 chia hết cho 3 và như
thế n2 và p2 cũng chia hết cho 3.
Hơn nữa 2a + b = 3a + (b − a) chia hết cho 3, suy ra a − b chia hết cho 3.
Tương tự thì b − c, c − a chia hết cho 3. Suy ra (a − b)(b − c)(c − a) chia hết
cho 27.

2. Tồn tại. Chẳng hạn có thể lấy a = 2, b = 0, c = 1.

Sau đây cũng là bài toán chia hết, nhưng ở mức độ khó hơn hẳn, đòi hỏi học sinh
phải có suy luận tốt và nắm chắc được nhiều kiến thức.

Ví dụ 2. (PTNK 2016 - CT) Cho x, y là hai số nguyên dương mà x2 + y 2 + 10


chia hết cho xy.

1. Chứng minh rằng x, y là hai số lẻ và nguyên tố cùng nhau.


x2 + y 2 + 10
2. Chứng minh k = chia hết cho 4 và k ≥ 12.
xy


Nhận xét. Bài toán này cũng giống bài toán trên, là liên quan đến các số chính
phương x2 , y 2 . Việc chứng minh chẵn lẻ liên quan đến số dư khi chia cho 4 của một
số chính phương.
Câu a) chỉ là bài toán xét trường hợp khá dễ nhìn, khi phản chứng là giả sử x, y
không cùng là số lẻ, từ đó khi xét tính chẵn lẻ của x2 + y 2 + 10 và xy sẽ giải quyết
được vấn đề.
Việc chứng minh nguyên tố cùng nhau thì cách tiếp cận quen thuộc nhất là gọi ước
chung lớn nhất và chứng minh nó bằng 1.
Câu b) khó hơn khi có hai ý, ý đầu có thể áp dụng tiếp câu a, nhưng ý sau việc
chứng minh k ≥ 12 có thể đánh lừa nhiều học sinh trong khi việc đơn giản chỉ là
chứng minh k chia hết cho 3 là giải quyết được bài toán, mà chứng minh k chia hết

Tập san Toán học STAR EDUCATION


NGUYỄN TĂNG VŨ

cho 3 cũng là việc xét số dư của tử và mẫu thức khi chia cho 3. Sau đây là lời giải
chi tiết.
Lời giải.

1. Giả sử trong hai số x, y có một số chẵn, vì vai trò x, y như nhau nên có thể giả
sử x chẵn. Suy ra x2 + y 2 + 10 chia hết cho 2, suy ra y chẵn. Khi đó x2 + y 2 + 10
chia hết cho 4, suy ra 10 chia hết cho 4 vô lý.
Vậy trong hai số đều là số lẻ.
Đặt d = (x, y), x = d.x0 , y = d.y 0 ta có x2 + y 2 + 10 = d2 (x02 + y 02 ) + 10 chia
hết cho d2 x0 y 0 . Suy ra 10 chia hết cho d2 . Suy ra d = 1. Vậy x, y nguyên tố
cùng nhau.

2. Đặt x = 2m + 1, y = 2n + 1, suy ra

4(m2 + m + n2 + n + 3
k= .
(2m + 1)(2n + 1)

Ta có 4, (2m + 1).(2n + 1) nguyên tố cùng nhau. Suy ra m2 + n2 + m + n + 3


chia hết cho (2m + 1)(2n + 1). Từ đó ta có k chia hết cho 4.
Chứng minh k ≥ 12 bằng hai cách.

Cách 1: Ta có x2 + y 2 + 10 = kxy. Nếu trong hai số x, y có một số chia hết


cho 3, giả sử x chia hết cho 3. Ta có y 2 + 10 chia hết cho 3 vô lý vì y 2 chia 3
dư 0 hoặc dư 1.

Vậy x, y không chia hết cho 3, suy ra x2 + y 2 + 10 chia hết cho 3 và 3, xy


nguyên tố cùng nhau. Do đó k chia hết cho 3. Do đó k chia hết cho 12, vậy
k ≥ 12.

Cách 2: Xét k = 4 ta có x2 + y 2 + 10 = 4xy (∗) ⇔ (x − 2y)2 = 3y 2 − 10.


Ta có (x − 2y)2 chia 3 dư 0 hoặc 1 mà 3y 2 − 10 chia 3 dư 2, nên phương trình
(*) không có nghiệm nguyên dương. Xét k = 8 ta có

x2 + y 2 + 10 = 8xy(∗) ⇔ (x − 4y)2 = 15y 2 − 10.

Ta có (x − 4y)2 chia 3 dư 0 hoặc 1 mà 15y 2 − 10 chia 3 dư 2 nên (**) không


có nghiệm nguyên dương.
Vậy k ≥ 12.

Sau chia hết, các kiến thức về phương trình nghiệm nguyên cũng rất quan trọng,
trong nhiều bài thi của PTNK kĩ năng giải phương trình nghiệm nguyên giúp mình
được nhiều việc.
Sau đây là bài toán số học, nhưng bản chất số học thì ít mà đại số thì nhiều, chỉ
việc biến đổi đại số vài dòng là xong. Tuy vậy nhiều học sinh sau khi đọc đề lại phát
hoảng, vì đề bài phát biểu nghe rất "kinh", đánh lừa được các thí sinh yếu bóng
vía. Bài toán sau chế tác từ bài thi của Bulgari:

Tập san Toán học STAR EDUCATION


6 NGUYỄN TĂNG VŨ

Ví dụ 3. (PTNK 2012 - CT) Số nguyên dương n được gọi là số điều hòa nếu
như tổng các bình phương của các ước của nó (kể cả 1 và n) đúng bằng (n + 3)2 .

1. Chứng minh rằng số 287 là số điều hòa.

2. Chứng minh rằng số n = p3 (p nguyên tố ) không phải là số điều hòa.

3. Chứng minh rằng nếu số n = pq (p, q là các số nguyên tố khác nhau) là số


điều hòa thì n + 2 là số chính phương.

Nhận xét. Bài toán đưa ra định nghĩa số điều hòa, nghe có vẻ ghê gớm nhưng thực
ra không có ý nghĩa mấy, hoặc không phù hợp với từ điều hòa hay dùng. Nhiều thí
sinh đọc đề mà thuộc dạng yếu bóng vía sẽ bỏ qua, ngay cả bỏ qua câu a rất dễ.
Tuy nhiên nếu đã hiểu định nghĩa, việc giải quyết các câu hỏi là điều khá dễ.

Lời giải.

1. Số n = 287 có các ước dương là 1, 7, 41, 287. Ta có 12 + 72 + 412 + 2872 =


(287 + 3)2 nên 287 là số điều hòa.

2. Các ước dương của n = p3 là 1, p, p2 , p3 . Giả sử n là số điều hòa, ta có (n+3)2 =


12 + p2 + p4 + p6 ⇔ p4 + p2 = 6p3 + 8. Suy ra p|8 mà p nguyên tố nên p = 2.
Thử lại thấy không thỏa, vậy n = p3 không phải là số điều hòa với mọi số
nguyên tố p.

3. Các ước dương của n = pq là 1, p, q, pq. Vì n là số điều hòa nên ta có

1 + p2 + q 2 + p2 q 2 = (pq + 3)2 ⇔ p2 + q 2 = 6pq + 8 ⇔ (p + q)2 = 4(pq + 2).

Do 4 là số chính phương nên pq + 2 = n + 2 cũng là số chính phương.

Sau đây là một bài khá đẹp, ý tưởng từ phương pháp lùi vô hạn trong giải phương
trình nghiệm nguyên, tuy vậy các phải có suy luận một chút khác biệt.

Ví dụ 4. (PTNK 2014 - CT)

1. Tìm các số nguyên a, b, c sao cho a + b + c = 0 và ab + bc + ac + 3 = 0.

2. Cho m là số nguyên. Chứng minh rằng nếu tồn tại các số nguyên a, b, c
khác 0 sao cho a + b + c = 0 và ab + bc + ac + 4m = 0 thì cũng tồn tại các
số nguyên a0 , b0 , c0 sao cho a0 + b0 + c0 = 0 và a0 b0 + b0 c0 + a0 c0 + m = 0.

3. Với k là số nguyên dương, chứng minh rằng không tồn tại các số nguyên
a, b, c khác 0 sao cho a + b + c = 0 và ab + bc + ac + 2k = 0.

Lời giải.

Tập san Toán học STAR EDUCATION


NGUYỄN TĂNG VŨ

1. Từ a + b + c = 0, ab + bc + ca = −3 ta có a2 + b2 + c2 = 6. Do a, b, c vai trò như


nhau nên ta có thể giả sử |a| ≥ |b| ≥ |c|. Khi đó 1 < |a| < 3. Suy ra |a| = 2,
suy ra a = 2 hoặc a = −2.
Với a = 2 thì b + c = −2, b2 + c2 = 2 giải ra được b = c = −1.Ta có có bộ
(2; −1; −1) và các hoán vị.
Với a = −2 thì b + c = 2, b2 + c2 = 2, giải ra được b = c = 1, ta có bộ (−2; 1; 1)
và hoán vị.

2. Ta có a + b + c = 0 chẵn (1)và ab + bc + ac = −4m chẵn.(2)


Nếu 3 số a, b, c đều lẻ, không thỏa (1).
Nếu có 1 chẵn, 2 lẻ thì không thỏa (2).
a b 0 c
Do đó 3 số a, b, c đều chẵn. Khi đó đặt a0 = , b0 = , c = thì a0 , b0 , c0 thỏa
2 2 2
đề bài.

3. Với k = 0 ta có a + b + c = 0, ab + bc + ac = −1 thì a2 + b2 + c2 = 2 (3) .


Không có bộ 3 số nguyên a, b, c khác 0 thỏa (3).
Với k = 1 thì a + b + c = 0, ab + bc + ac = −2 khi đó a2 + b2 + c2 = 4 (4).
Giả sử |a| nhỏ nhất khi đó 1 ≤ a2 < 2 (không có a thỏa). Không tồn tại a, b, c
nguyên khác 0 thỏa (4).
Với k > 1.
Nếu k chẵn, đặt k = 2n ta có a + b + c = 0, ab + bc + ac + 4n = 0, theo câu b),
tồn tại a1 , b1 , c1 nguyên thỏa a1 + b1 + c1 = 0, a1 b1 + a1 c1 + b1 c1 + 4n−1 = 0.
Tương tự ta sẽ được an , bn , cn nguyên thỏa

an + bn + cn = 0, an bn + bn cn + an cn = −1 (vô nghiệm).

Nếu k lẻ đặt k = 2n + 1 ta có a + b + c = 0, ab + bc + ac + 2.4n = 0, làm tương


tự trên ta được

an + bn + cn = 0, an bn + bn cn + an cn = −2(vô nghiệm).

Vậy không tồn tại các số a, b, c khác 0 thỏa đề bài.

Ngoài ra việc sử dụng đồng dư cũng được khai thác qua các bài toán chia hết hoặc
các bài toán phương trình nghiệm nguyên, nhiều khi được sử dụng một cách bất
ngờ cũng gây khó khăn cho thí sinh và rất ít thí sinh làm trọn vẹn, sau đây là một
ví dụ:

Ví dụ 5. (PTNK 2018 - CT)


Cho An = 2018n + 2032n − 1964n − 1984n với n là số tự nhiên.

1. Chứng minh với mọi số tự nhiên n thì An chia hết cho 51.

2. Tìm tất cả những số tự nhiên n sao cho An chia hết cho 45.

Nhận xét Đây là dạng toán khá quen thuộc với học sinh, chỉ là việc xét các trường
hợp một cách khéo léo và cẩn thận để giải quyết bài toán.

Tập san Toán học STAR EDUCATION


8 NGUYỄN TĂNG VŨ

Lời giải.
1. Do 2018 ≡ 1964 (mod 3) ⇒ 2018n ≡ 1964n (mod 3).
2032 ≡ 1984 (mod 3) ⇒ 2032n ≡ 1984n (mod 3).
.
⇒ An .. 3.
Ta lại có 2018 ≡ 1984 (mod 17) ⇒ 2018n ≡ 1984n (mod 17).
2032 ≡ 1964 (mod 17) ⇒ 2032n ≡ 1964n (mod 17).
.
⇒ An .. 17.
.
Do (3; 17) = 1 nên An .. 51 ∀n

2. An = 2018n + 2032n − 1964n − 1984n .


.
ˆ Ta xét các trường hợp của n để An .. 5. Ta có

An ≡ (−2)n + 2n − 2 · (−1)n (mod 5).

Do đó nếu n lẻ ⇒ An ≡ 2 (mod 5) (loại).


Nếu n = 4k ⇒ An ≡ 2 · 24k − 2 ≡ 2 − 2 ≡ 0 (mod 5) (nhận)
Nếu n = 4k + 2 ⇒ An ≡ 2 · 24k+2 − 2 ≡ 8 − 2 ≡ 6 (mod 5) (loại).
. .
Vậy An .. 5 ⇔ n .. 4.
.
ˆ Ta xét các trường hợp của n để An .. 9. Ta có

An ≡ 2n + (−2)n − 2n − 4n (mod 9)
≡ 2n − 4n (mod 9) (Do n chẵn).
≡ 2n (1 − 2n ) (mod 9)
.
Vì (2; 9) = 1 ⇒ 2n − 1 .. 9.
Xét n = 3k với k ∈ N thì

An ≡ 23k − 1 ≡ (−1)k − 1 (mod 9) ⇒ 2|k.

Xét n = 3k + 1 với k ∈ N. Ta có

An ≡ 23k+1 − 1 ≡ 2 · (−1)k − 1 (mod 9) (loại).

Xét n = 3k + 2 với k ∈ N. Ta có

An ≡ 23k+2 − 1 ≡ 4 · (−1)k − 1 (mod 9) (loại).


. .
Vậy An .. 45 ⇔ n .. 12.

Nhận xét. Tóm lại bài toán số học thi vào lớp 10 Chuyên Toán luôn là bài toán
khó, nhưng không phải không kiếm được điểm, chỉ cần thí sinh bình tĩnh vận dụng
được kiến thức đã học có thể giải quyết được các ý a, ý b thì phức tạp hơn đòi hỏi
phải phân tích và xử lí khéo léo cẩn thận hơn, âu cũng hợp lí cho đề thi chọn học
sinh có năng khiếu toán.
Sau đây có một số bài tập cho các em rèn luyện trước kì thi cam go này.

Tập san Toán học STAR EDUCATION


NGUYỄN TĂNG VŨ

2. Bài tập rèn luyện

Bài 1. (Tuyển sinh vào lớp 10 Chuyên Toán trường PTNK 1997)

1. Tìm tất cả các số nguyên dương n sao cho n2n + 3n chia hết cho 5.

2. Tìm tất cả các số nguyên dương n sao cho n2n + 3n chia hết cho 25.

Lời giải.

1. a) Nếu n ta có 2n + 3n chia hết cho 5.


Xét n = 2k + 1 ta có n.2n + 3n = (n − 1)2n + 2n + 3n chia hết cho 5 khi và chỉ
khi n − 1 chia hết cho 5, hay k chia hết cho 5,suy ra k = 5q. Vậy n = 10q + 1.
Xét n = 2k ta có n.2n + 3n = 2k.4k + 9k = 2k.4k + 4k + 9k − 4k = (2k + 1).4k +
9k − 4k chia hết cho 5 khi 2k + 1 chia hết cho 5. Khi đó k = 5q + 2, suy ra
n = 10q + 4. Vậy với n = 10q + 1, 10q + 4 thì n.2n + 3n chia hết cho 5.

2. ˆ Theo câu a để A = n.2n + 3n chia hết cho 5 thì n = 10q + 1, 10q + 4. Ta


tìm q để n.2n + 3n chia hết cho 25.
ˆ Với n = 10q + 1 ta có

A = (10q + 1)210q+1 + 310q+1 = (20q + 2).1024q + 3.310q .

Ta có 1024 ≡ −1 (mod 25), 310 ≡ −1 (mod 25). Suy ra

A ≡ (20q + 2)(−1)q + 3.(−1)q (mod 25)

hay
A = (−1)q (20q + 5) (mod 25).
Suy ra A chia hết cho 25 khi và chỉ khi 20q + 5 chia hết cho 25 hay 4q + 1
chia hết cho 5. Suy ra q = 5k + 1. Vậy n = 10(5k + 1) + 1 = 50k + 11.
ˆ Với n = 10q + 4. Ta có

A = (10q + 4)210q+4 + 310q+4 = (160q + 64)210q + 81.310q


≡ (10q + 14)(−1)q + 6(−1)q (mod 25) ≡ (−1)q (10q + 20) mod 25.

Do đó A chia hết cho 25 khi và chỉ khi 10q + 20 chia hết cho 25 hay q + 2
chia hết cho 5, suy ra q = 5k + 3. Suy ra n = 10(5k + 3) + 4 = 50k + 34.
Vậy n = 50k + 11, 50k + 34 với k nguyên dương.

Bài 2. (Tuyển sinh vào lớp 10 Chuyên Toán trường PTNK 1997)

1. Tìm tất cả các số nguyên dương sao cho 2n − 1 chia hết 7.

2. Cho số nguyên tố p ≥ 5. Chứng minh rằng A = 3p − 2p − 1 chia hết cho 42p.

Tập san Toán học STAR EDUCATION


10 NGUYỄN TĂNG VŨ

Lời giải.

1. Ta xét các trường hợp sau

ˆ TH1: n = 3k ta có 2n − 1 = 23k − 1 = 8k − 1 chia hết cho 7.


ˆ TH2: n = 3k + 1 ta có 2n − 1 = 2.8k − 1 chia 7 dư 1.
ˆ TH3: n = 3k + 2 ta có 2n − 1 = 4.8k − 1 chia 7 dư 3.
ˆ Vậy 2n − 1 chia hết cho 7 khi và chỉ khi n chia hết cho 3.

2. Ta có phân tích 42p = 2.3.7.p. Khi đó, xét các trường hợp sau:

ˆ TH1: p = 7 ta có 37 − 27 − 1 chia hết cho 42.7.

ˆ TH2: p > 7 khi đó các số 2, 3, 7, p đôi một nguyên tố cùng nhau.


+ Ta có 3p − 1 − 2p chia hết cho 2.
+ 2p + 1 chia hết cho 3 vì p lẻ, suy ra 3p − 2p − 1 chia hết cho 3.
+ p nguyên tố lớn hơn hoặc bằng 5, suy ra p = 6k + 1 hoặc p = 6k + 5.
Nếu p = 6k + 1 ta có 3p − 2p − 1 = 36k+1 − 26k+1 − 1 = 3.36k − 2.26k − 1.
Ta có 36 ≡ 1 (mod 7), suy ra 36k ≡ 1 (mod 7), tương tự thì 26k ≡ 1
(mod 7). Do đó 3.36k − 2.26k − 1 ≡ 0 (mod 7).
Nếu p = 6k + 5 ta có 3p − 2p − 1 ≡ 35 − 25 − 1 ≡ 0 (mod 7).
Do đó 3p − 2p − 1 chia hết cho 7.
+ Theo định lý Fermat nhỏ, ta có 3p ≡ 3( mod p), 2p ≡ 2 (mod 7). Suy
ra 3p − 2p − 1 chia hết cho p.
Vậy 3p − 2p − 1 chia hết cho 42p.

Bài 3. Cho n là số tự nhiên. Chứng minh rằng 3n n3 + 1 chia hết cho 7 khi và
chỉ khi 3n + n3 chia hết cho 7.

Lời giải.
Nếu 3n n3 + 1 chia hết cho 7. Suy ra n không chia hết cho 7, suy ra n6 − 1 chia
hết cho 7.
Ta có n3 (3n + n3 ) = n3 3n + n6 = n3 3n + 1 + n6 − 1 chia hết cho 7.
Mà (n, 7) = 1. Suy ra 3n + n3 chia hết cho 7.

ˆ Nếu 3n + n3 chia hết cho 7. Làm tương tự ta cũng có n3 3n + 1 chia hết cho 7.

Bài 4. Chứng minh rằng

1. Trong 5 số nguyên thì có 3 số có tổng chia hết cho 3.

2. Trong 17 số nguyên thì có 9 số có tổng chia hết cho 9.

Tập san Toán học STAR EDUCATION


NGUYỄN TĂNG VŨ

Lời giải.
Một số khi chia cho 3 có các số dư là 0, 1, 2.
Nếu trong 5 số khi chia cho 3 số có đủ 3 số dư 0, 1, 2 thì tổng 3 số này chia
hết cho 3.
Nếu có 2 loại số dư thì có 3 số khi chia cho 3 có cùng một số dư, tổng của
chúng chia hết cho 3.
Nếu có 1 loại số dư, thì tổng 3 số bất kì đều chia hết cho 3.
2. Đặt các số đó là a1 , a2 , · · · , a16 , a17 .
1.
Trong 5 số a1 , · · · , a5 có 3 số có tổng chia hết cho 3, không mất tính tổng quát
là a1 , a2 , a3 . Đặt a1 + a2 + a3 = 3b1 .
Trong 5 số a4 , · · · , a8 có 3 số có tổng chia hết cho 3, giả sử a4 , a5 , a6 và đặt
a4 + a5 + a6 = 3b2 .
Tương tự ta xây dựng được các số b3 , b4 , b5 .
Khi đó áp dụng tiếp cho 5 số b1 , b2 , b3 , b4 , b5 có 3 số có tổng chia hết cho 3, giả
sử b1 , b2 , b3 có tổng chia hết cho 3. Khi đó 9 số a1 , · · · , a9 có tổng chia hết cho
9. Bài toán được giải quyết.

Bài 5. Tìm tất cả các số tự nhiên x, y thỏa: 5x = y 4 + 4y + 1

Lời giải. Trước hết, ta thấy có một nghiệm là (0; 0).


ˆ Dễ thấy y chẵn nên y 4 + 4y + 1 ≡ 1( mod 8). Suy ra x chẵn, x = 2k. Khi đó
(5k )2 = y 4 + 4y + 1 là số chính phương.
ˆ Ta có y ≥ 1 nên y 4 < y 4 + 4y + 1 < (y 2 + 2)2 . Suy ra

y 4 + 4y + 1 = (y 2 + 1)2 ⇔ y = 2,
suy ra x = 2.
Vậy có 2 cặp nghiệm (0; 0), (2; 2).

Bài 6. Chứng minh rằng phương trình y 2 + y = x + x2 + x3 không có nghiệm


nguyên dương.

Lời giải. Ta có x3 = (y − x)(y + x + 1).


ˆ Gọi d là ước nguyên tố chung lớn nhất của y − x, y + x + 1, nếu d là số nguyên
tố thì d|x, d|y, suy ra d|1 (vô lý), Vậy y − x, y + x + 1 nguyên tố cùng nhau.
ˆ Do đó y − x = a3 , y + x + 1 = b3 , ab = x.

ˆ Ta có phương trình b3 − a3 = 2ab + 1 với a, b nguyên dương và b > a ≥ 1. Ta


có b3 − a3 ≥ a2 + b2 + ab > 2ab + 1.
ˆ Vậy phương trình không có nghiệm trong tập các số nguyên dương.

Tập san Toán học STAR EDUCATION


12 NGUYỄN TĂNG VŨ

Bài 7. Cho các số nguyên dương a, b, c, d thỏa ab = cd. Chứng minh rằng a + b +
c + d là hợp số.

.
Lời giải. Đặt k = (a, c), a = ka0 , c = kc0 , Suy ra a0 b = c0 d, suy ra b..c0 , đặt b = mc0 ,
suy ra d = ma0 .
Khi đó a + b + c + d = ka0 + mc0 + kc0 + ma0 = (k + m)(a0 + c0 ) là hợp số.

Bài 8. Chứng minh rằng nếu x2 + 2y là một số chính phương với x, y nguyên
dương thì x2 + y là tổng của hai số chính phương.

Lời giải. Đặt x2 + 2y = k 2 . Suy ra 2y = (k − x)(k + x). Suy ra k, x cùng tính chẵn
lẻ. Ta xét các trường hợp sau
ˆ Nếu k = 2m, x = 2n thì y = 2(m − n)(m + n).
Khi đó x2 + y = 2n2 + 2(m − n)(m + n) = 2m2 = m2 + m2 .

ˆ Nếu k = 2m + 1, x = 2n + 1 thì y = 2(m − n)(m + n + 1).


x2 +y = (2n+1)2 +2(m−n)(m+n+1) = 4n2 +4n+1+2m2 −2n2 +2m−2n =
2n2 + 2n + 2m2 + 2m + 1 = (m + n + 1)2 + (m − n)2 .

Bài 9. Chứng minh rằng nếu tích của hai số nguyên tố cùng nhau là một số
chính phương thì mỗi số cũng là số chính phương.

Lời giải. Cho ab = x2 , trong đó (a, b) = 1.


Đặt d = (a, x), a = a0 d, x = x0 d ta có a0 b = x02 d.
Do (a0 , x02 ) = 1 nên b chia hết cho x02 .
Mặt khác do (a, b) = 1 nên (b, d) = 1, suy ra x02 chia hết cho b.
Do đó b = x02 , a0 = d. Từ đó ta có a = a02 , b = x02 là các số chính phương.
Nhận xét. Tương tự nếu (a, b) = 1 và ab = xk thì a, b là lũy thừa bậc k của một
số nguyên. Đây là một bổ đề rất hay sử dụng.

Bài 10. Cho các số nguyên dương a, b thỏa 2a2 + a = 3b2 + b.

1. Tìm a, b biết a và b là hai số nguyên tố cùng nhau.

2. Chứng minh a − b và 2a + 2b + 1 là các số chính phương.

Lời giải. 1. a(2a + 1) = b(3b + 1). Ta có 3b + 1 chia hết cho a và 2a + 1 chia hết
cho b.
Đặt 2a + 1 = kb, suy ra 3b + 1 = ka. Suy ra 6ab + 2a + 3b + 1 = k 2 ab, suy ra
k = 1, 2.
Nếu k = 1 ta có 2a + 1 = b, 3b + 1 = a (Vô nghiệm).
Nếu k = 2 ta có 2a + 1 = 2b, 3b + 1 = 2a. (Vô nghiệm). Phương trình vô
nghiệm.

Tập san Toán học STAR EDUCATION


NGUYỄN TĂNG VŨ

2. Ta có (a − b)(2a + 2b + 1) = b2 .
Giả sử p là ước nguyên tố của a − b, 2a + 2b + 1, suy ra p|b2 ⇒ p|b, suy ra p|a,
suy ra p|1 (vô lý).
Do đó (a − b, 2a + 2b + 1) = 1.
Từ đó ta có a − b, 2a + 2b + 1 là các số chính phương.

Bài 11. Số tự nhiên n được gọi là số đẹp nếu tồn tại các số tự nhiên x, y, z sao
cho: n = [x; y] + [y; z] + [z; x] với [a; b] là bội chung nhỏ nhất của hai số a, b

1. Chứng minh rằng n = 2017 là số đẹp.

2. Chứng minh rằng mọi số lẻ đều là số đẹp.

3. Chứng minh rằng n = 22017 không phải là số đẹp.

Lời giải. 1. Ta chọn (x; y; z) = (1, 1, 1008). Khi đó: [1; 1008] + [1; 1008] + [1; 1] =
1008 + 1008 + 1 = 2017.
2. Với mọi số n lẻ, tức là: n = 2k + 1 trong đó k là số nguyên dương. Ta chọn:
(x; y; z) = (1, 1, k), khi đó: [1; k] + [1; k] + [1; 1] = k + k + 1 = 2k + 1 = n. Vậy
mọi số lẻ đều là số đẹp.
3. Do 22017 là số chẵn nên chỉ có hai trường hợp xảy ra, hoặc cả ba số
[x; y] , [y; z] , [z; x]
đều là số chẳn, hoặc trong ba số này có hai số lẻ và một số chẳn. Giả sử tồn
tại hai số lẻ, giả sử: [x; y] lẻ, suy ra: x, y đều là số lẻ, [y; z] lẻ, suy ra: y, z đều
là số lẻ, từ đó ta có: [z; x] cũng là số lẻ, như vậy: [x; y] + [y; z] + [z; x] là số
lẻ (mâu thuẫn). Từ đó ta có: [x; y] , [y; z] , [z; x] phải là ba số chẳn. Như vậy
trong ba số x, y, z phải có ít nhất hai số chẳn, giả sử là x và y.

Đặt x = 2a t1 (t1 là số tự nhiên lẻ), y = 2b t2 (t2 là số tự nhiên lẻ). Không mất


tính tổng quát giả sử 2017 ≥ a ≥ b ≥ 0. Ta xét hai trường hợp:
(a) z lẻ. Khi đó ta có: [x; y] = 2a m1 với m1 là số lẻ, [y; z] = 2b m2 , với m2 là
số lẻ, [z; x] = 2a m3 , với m2 là số lẻ. Từ đó
2a m1 + 2b m2 + 2a m3 = 22017 ⇔ 2b 2a−b m1 + m2 + 2a−b m3 = 22017


⇔ 2a−b m1 + m2 + 2a−b m3 = 22017−b .

Do vế trái của đẳng thức là một số lẻ, vế phải là một số chẳn. Từ đó ta


có trường hợp này không thể xảy ra.
(b) z là số chẳn. Như vậy, x, y, z đều là số chẳn, đặt: z = 2c t3 , với (t3 là số
tự nhiên lẻ) không mất tính tổng quát, giả sử: 2017 ≥ a ≥ b ≥ c ≥ 0.
Vậy: [x; y] = 2a m1 , [y; z] = 2b m2 , [z; x] = 2a m3 với m1 , m2 , m3 là ba số
tự nhiên lẻ. Từ đó
2a m1 + 2b m2 + 2a m3 = 22017 ⇔ 2b 2a−b m1 + m2 + 2a−b m3


= 22017 ⇔ 2a−b m1 + m2 + 2a−b m3 = 22017−b .

Tập san Toán học STAR EDUCATION


14 NGUYỄN TĂNG VŨ

Do vế trái của đẳng thức là một số lẻ, vế phải là một số chẳn. Từ đó ta


có trường hợp này không thể xảy ra.

Bài 12. Chứng minh rằng không tồn tại các số nguyên a, b thỏa đồng thời các
điều kiện sau:

1. 16a − 9b là số nguyên tố.

2. ab là số chính phương.

3. a + b cũng là số chính phương.

Lời giải. Giả sử tồn tại a, b thỏa đề bài. Đặt ab = x2 , a + b = y 2 .


Gọi d là ước chung lớn nhất của a và b. Giả sử d > 1. Đặt a = a0 d, b = b0 d, suy ra
a0 , b0 nguyên tố cùng nhau.
Ta có 16a − 9b = d(16a0 − 9b0 ) nguyên tố, suy ra 16a0 − 9b0 = 1 và d nguyên tố.
Ta có d2 a0 b0 = x2 , suy ra a0 b0 chính phương mà (a0 , b0 ) = 1 nên a0 , b0 là các số chính
phương. (1)
Ta có 16a0 = 9b0 + 1 ≡ 0( mod 4), suy ra b0 ≡ 3( mod 4) không thể là số chính
phương. (mâu thuẫn với (1)).
Vậy d = 1 hay a, b nguyên tố cùng nhau.
Từ ab = x2 ta có a, b chính phương. Đặt a = m2 , b = n2 (m, n > 0), suy ra

16a − 9b = 16m2 − 9n2 = (4m − 3n)(4m + 3n)

là số nguyên tố. Suy ra 4m − 3n = 1. Khi đó m = 3k + 1, n = 4k + 1. Suy ra

y 2 = a + b = (3k + 1)2 + (4k + 1)2 = 25k 2 + 14k + 2 (2).

Ta có
(5k + 1)2 < 25k 2 + 14k + 2 < (5k + 2)2
nên (2) vô nghiệm.
Vậy không tồn tại a, b thỏa đề bài.

Tập san Toán học STAR EDUCATION


Một số vấn đề cơ bản về Dãy số

Nguyễn Công Thành


(SV trường ĐHSP TPHCM)

Tài liệu này trình bày một số vấn đề cơ bản về Dãy số nhằm giúp học sinh nắm được
những kiến thức nền tảng thiết yếu về Dãy số, phục vụ cho việc tiếp cận những vấn đề
chuyên sâu hơn về Dãy số sau này.

1. Phương pháp quy nạp toán học


1.1. Lý thuyết
Quy nạp là quá trình rút ra nguyên lý chung từ sự quan sát một loạt những sự vật,
sự kiện riêng lẻ. Nó được sử dụng rỗng rãi trong các ngành khoa học như Vật lí,
Hóa học, Sinh học, . . . Thậm chí ngay cả trong Toán học, ngành được coi là được
xây dựng dựa trên những lí luận chặt chẽ, quy nạp cũng rất phổ biến. Rất nhiều
kết quả toán học được tìm ra dựa trên những quan sát, nhận xét trong trường hợp
nhỏ lẻ. Tuy nhiên, từ những trường hợp riêng ấy đi đến tính đúng đắn của kết quả
tổng quát là không tầm thường, cần được kiếm tra, chứng minh bằng các lập luận
chặt chẽ. Phương pháp quy nạp toán học cho phép chúng ta thực hiện điều đó.
Trước tiên, ta đến với định lý sau

Định lý 1 (Nguyên lí sắp thứ tự tốt). Mọi tập con khác rỗng S của N đều có
phần tử nhỏ nhất.

Chứng minh. Vì S 6= ∅ nên ∃m ∈ S. Xét T = {n ∈ S : n 6 m} thì T khác rỗng và


chỉ có hữu hạn phần tử nên có phần tử nhỏ nhất là k. Rõ ràng k cũng là phần tử
nhỏ nhất của S vì k ∈ S và với mỗi phần tử n ∈ S, nếu n ∈ T thì n > k và nếu
n 6∈ T thì n > m > k.

Định lý 2 (Nguyên lí quy nạp toán học). Cho S là tập con của N, thỏa mãn

(i) 1 ∈ S.

(ii) Với mỗi n ∈ N, nếu n ∈ S thì n + 1 ∈ S.

Khi đó S = N.

15
16 NGUYỄN CÔNG THÀNH

Chứng minh. Đặt T = {n ∈ N : n 6∈ S} . Ta sẽ chứng minh T là tập rỗng, từ đó suy


ra điều cần chứng minh.
Thật vậy, giả sử phản chứng T khác rỗng. Khi đó theo nguyên lí sắp thứ tự tốt,
T có phần tử nhỏ nhất là k. Chú ý rằng vì 1 ∈ S nên k > 2, do đó k − 1 ∈ S
vì tính nhỏ nhất của k trong T . Tuy nhiên, lúc này từ tính chất (ii) của S, ta có
(k − 1) + 1 = k ∈ S, vô lí.
Vậy điều đã giả sử là sai, từ đó suy ra điều cần chứng minh.

Nhận xét.

ˆ Trong định lý trên, nếu ta giữ nguyên giả thiết (ii) và thay giả thiết (i) bởi giả
thiết (i’) như sau

(i’) k ∈ S, với k là số tự nhiên cho trước

thì kết luận sẽ là {n ∈ N : n > k} ⊂ S.

ˆ Trong định lý trên, nếu ta giữ nguyên giả thiết (i) và thay giả thiết (ii) bởi giả
thiết (ii’) như sau

(ii’) Với mỗi n ∈ N, nếu {1, 2, . . . , n} ⊂ S thì n + 1 ∈ S

thì kết luận vẫn là S = N. Nguyên lí tương ứng được gọi là Nguyên lí quy
nạp mạnh.

Từ các nguyên lí quy nạp, ta xây dựng được phương pháp quy nạp toán học. Cụ
thể, muốn chứng minh mệnh đề P (n) đúng với mọi số tự nhiên n, ta thực hiện hai
bước sau

1. Chỉ ra P (1) đúng (bước cơ sở).

2. Chứng minh với mọi số tự nhiên n, nếu P (n) đúng thì P (n + 1) cũng đúng
(bước quy nạp).

Khi đó, tập S = n ∈ N : P (n) đúng thỏa mãn điều kiện của nguyên lí quy nạp
nên S = N.
Một cách tương tự, ta cũng xây dựng được phương pháp quy nạp mạnh dựa trên
nguyên lí quy nạp mạnh, xin dành lại cho bạn đọc.
Ta xem xét một số ví dụ sau

Ví dụ 1. Chứng minh rằng với mọi số nguyên dương n, ta có

n2 (n + 1)2
13 + 23 + · · · + n3 = .
4


Chứng minh. Với n = 1, dễ thấy đẳng thức trong đề bài là đúng. Giả sử

3 3 3n2 (n + 1)2
1 + 2 + ··· + n = .
4

Tập san Toán học STAR EDUCATION


NGUYỄN CÔNG THÀNH

Khi đó ta có
n2 (n + 1)2 (n + 1)2 (n + 2)2
13 + 23 + · · · + (n + 1)3 = + (n + 1)3 = .
4 4
Vậy theo nguyên lí quy nạp, ta suy ra điều cần chứng minh.

Ví dụ 2. Chứng minh rằng với mọi số tự nhiên n > 4, ta có n! > 2n . 

Chứng minh. Với n = 4, ta có 4! = 24 > 16 = 24 nên bất đẳng thức cần chứng minh
đúng với n = 4. Giả sử n! > 2n , n > 4, khi đó ta có
(n + 1)! = n! · (n + 1) > 2n · 2 = 2n+1 .
Từ đó theo nguyên lí quy nạp, ta có điều cần chứng minh.

Ví dụ 3 (Bất đẳng thức Bernoulli). Chứng minh rằng với mọi a > −1, với mọi
n ∈ N, ta có (1 + a)n > 1 + na. 

Chứng minh. Lấy a > −1 tùy ý. Với n = 1, hiển nhiên bất đẳng thức trong đề bài
đúng. Giả sử (1 + a)n > 1 + na, khi đó ta có
(1 + a)n+1 > (1 + na) · (1 + a) = 1 + (n + 1) a + na2 > 1 + (n + 1) a.
Theo nguyên lí quy nạp, ta có điều cần chứng minh.

1.2. Bài tập

Bài 13. Chứng minh rằng với mọi số tự nhiên n, ta có


n (n + 1) (2n + 1) (3n2 + 3n − 1)
a) 14 + 24 + · · · + n4 = .
30
b) 1 · 4 + 2 · 7 + · · · + n (3n + 1) = n (n + 1)2 .
1 2 n n+2
c) + + ··· + n = 2 − n .
2 4 2 2
(−1)n−1 n (n + 1)
d) 12 − 22 + 32 − · · · + (−1)n n2 = .
2

Bài 14. Chứng minh rằng với mọi số tự nhiên n, ta có


4n (2n)!
a) < .
n+1 (n!)2
1 1 √
b) 1 + √ + · · · √ < 2 n.
2 n

Tập san Toán học STAR EDUCATION


18 NGUYỄN CÔNG THÀNH

1 1 1
c) 1 + 2
+ 2 + · · · + 2 < 2.
2 3 n
1 3 2n − 1 1
d) · ··· <√ .
2 4 2n 2n + 1
1 2 2n 1 2n+1
e) + + ··· + = + .
1 + x 1 + x2 1 + xn x − 1 1 − x2n+1

Bài 15. Chứng minh rằng với mọi số tự nhiên n, ta có


 n
n+1
a) 6 n + 1.
n
n
an + b n

a+b
b) 6 , trong đó a, b > 0.
2 2

Bài 16. Cho n số nguyên dương x1 , x2 , . . . , xn , với n > 2. Chứng minh rằng

(1 + x1 ) (1 + x2 ) · · · (1 + xn ) > 1 + x1 + x2 + · · · + xn .

Bài 17. Chứng minh rằng với mọi số tự nhiên n, ta có

1. 11 | 62n + 10 · 3n .

2. 5 | 7 · 22n−2 + 32n−1 .

3. 7 | 32n+1 + 2n+2 .

4. 14 | 6n + 8n khi n lẻ.

1 1
Bài 18. Cho số thực x thỏa mãn x + là số nguyên. Chứng minh rằng xn + n
x x
cũng là số nguyên với mọi số tự nhiên n.

n5 n4 n3 n
Bài 19. Chứng minh + + − luôn là số tự nhiên với mọi số tự nhiên
5 2 3 30
n.

Bài 20. Chứng minh rằng với mọi số tự nhiên n, ta có

Tập san Toán học STAR EDUCATION


NGUYỄN CÔNG THÀNH

nx (n + 1) x
sin · sin
a) sin x + sin 2x + · · · + sin nx = 2 2 .
x
sin
2
nx (n + 1) x
cos · sin
b) cos x + cos 2x + · · · + cos nx = 2 2 .
x
sin
2

Bài 21 (Bất đẳng thức Cauchy - Schwarz). Cho các số thực a1 , a2 , . . . , an và


b1 , b2 , . . . , bn bất kì. Chứng minh rằng
n
! n ! n
!2
X X X
a2k b2k > ak b k .
k=1 k=1 k=1

Bài 22 (Bất đẳng thức AM - GM). Cho các số thực dương a1 , a2 . . . , an bất kì.
Chứng minh rằng
a1 + a2 + · · · + an √
> n a1 a2 · · · an .
n

Bài 23. Chứng minh rằng với mọi số nguyên dương k, tồn tại số nguyên dương
n sao cho 2k n + 17 là số chính phương.

Bài 24. Cho a1 , a2 , . . . là dãy các số thực thỏa mãn ai + aj 6 ai+j với mọi số
nguyên dương i, j. Chứng minh rằng
a2 a3 an
a1 + + + ··· > an
2 3 n
với mọi số nguyên dương n.

2. Dãy số
2.1. Lý thuyết

Định nghĩa 1. Dãy số thực là một ánh xạ f : N → R.


Thông thường, ta viết xn = f (n) và kí hiệu dãy số trên là
x1 , x2 , . . . , xn , . . . hoặc (xn )n∈N .
Nếu không có gì nhầm lẫn, ta còn có thể viết (xn ) thay vì (xn )n∈N và gọi (xn ) là dãy
thay vì dãy số. Ta gọi xn là số hạng thứ n, và nếu xn có thể biểu diễn bởi một công
thức đại số thì công thức đó được gọi là công thức tổng quát của dãy (xn ).

Tập san Toán học STAR EDUCATION


20 NGUYỄN CÔNG THÀNH

Một dãy số thường được xác định bởi các cách dưới đây

ˆ Dãy số được xác định bằng một mệnh đề mô tả các số hạng liên tiếp của nó.

ˆ Dãy số được xác định bằng công thức tổng quát.

ˆ Dãy số được xác định bằng công thức truy hồi: Trước hết, cho một (hoặc một
vài) số hạng đầu tiên, sau đó cho công thức biểu diễn số hạng thứ n qua số
hạng (hoặc một vài số hạng) đứng trước nó.

Định nghĩa 2.

ˆ Dãy số (xn ) được gọi là dãy tăng (giảm) nếu với mọi số tự nhiên n, ta có
xn < xn+1 (xn > xn+1 ).

ˆ Dãy số (xn ) được gọi là dãy không tăng (không giảm) nếu với mọi số tự
nhiên n, ta có xn > xn+1 (xn 6 xn+1 ).

Dãy số sở hữu các tính chất trên được gọi chung là dãy đơn điệu.

Định nghĩa 3.

ˆ Dãy số (xn ) được gọi là bị chặn trên nếu tồn tại số thực M sao cho xn 6 M
với mọi số tự nhiên n.

ˆ Dãy số (xn ) được gọi là bị chặn dưới nếu tồn tại số thực m sao cho xn > m
với mọi số tự nhiên n.

ˆ Dãy số (xn ) được gọi là bị chặn nếu nó vừa bị chặn trên, vừa bị chặn dưới.

Định nghĩa 4. Dãy số (xn ) được gọi là tuần hoàn nếu tồn tại số nguyên dương
T sao cho xn+T = xn với mọi số tự nhiên n.
Số nguyên dương T nói trên được gọi là chu kì tuần hoàn của (xn ).

Ta thấy rõ ràng nếu T là chu kì của (xn ) thì kT cũng là chu kì của (xn ), trong đó
k là số nguyên dương tùy ý.

2.2. Bài tập

Bài 25. Cho dãy số (xn ) thỏa mãn x0 = 0 và

xn + xn−2 > 2xn−1

với mọi số tự nhiên n > 2. Với số nguyên dương n bất kì, chứng minh rằng
nxk 6 kxn với mọi số tự nhiên k thỏa mãn 0 6 k 6 n.

Tập san Toán học STAR EDUCATION


NGUYỄN CÔNG THÀNH

Bài 26. Cho (xn ) là dãy số thực dương thỏa mãn

x2n 6 xn − xn+1

với mọi số nguyên dương n. Chứng minh rằng xn < 1/n với mọi số nguyên dương
n.

Bài 27. Cho dãy (xn ) xác định bởi x1 = 1, x2 = 2 và


xn xn−1 + 1
xn+1 =
xn−1

với mọi số tự nhiên n > 2. Chứng minh rằng xn > 2n với mọi số tự nhiên n > 3.

Bài 28. Cho dãy số (xn ) thỏa mãn x1 = 2, x2 = 500, x3 = 2000 và


xn+2 + xn+1 xn+1
=
xn+1 + xn−1 xn−1

với mọi số tự nhiên n > 2. Chứng minh rằng (xn ) là dãy số nguyên, đồng thời
2n | xn với mọi số nguyên dương n.

Bài 29. Cho dãy (xn ) xác định bởi x1 = 1 và


1
xm+n + xm−n = (x2m + x2n )
2
với mọi số tự nhiên m, n thỏa mãn m > n. Xác định công thức tổng quát của
(xn ).

Bài 30. Cho trước số nguyên dương n. Xét dãy các số a0 , a1 , a2 , . . . , an thỏa
mãn a0 = 1/2 và
a2
ak = ak−1 + k−1
n
với mọi k = 1, 2, . . . , n. Chứng minh rằng 1 − 1/n < an < 1.

Bài 31. Cho dãy số (xn ) xác định bởi x1 = 1 và


xn n
xn+1 = +
n xn

với mọi n > 1. Chứng minh rằng bx2n c = n với mọi số tự nhiên n > 4.

Tập san Toán học STAR EDUCATION


22 NGUYỄN CÔNG THÀNH

Bài 32. Cho f (x) là đa thức hệ số nguyên và dãy (xn ) xác định bởi x0 = 0 và
xn+1 = f (xn ) với mọi số tự nhiên n. Chứng minh rằng nếu (xn ) tuần hoàn thì
chu kì nhỏ nhất của nó không vượt quá 2.

Bài 33. Cho dãy số (xn ) xác định bởi x1 = 2 và

xn+1 = x2n − xn + 1

với mọi số nguyên dương n. Chứng minh rằng với mọi số nguyên dương n > 1, ta

1 1 1 1 1
1 − 2n−1 < + + ··· + < 1 − 2n .
2 x1 x 2 xn 2

√ (xn ). Chứng minh rằng tồn tại vô số số nguyên


Bài 34. Cho dãy số thực dương
dương n thỏa mãn 1 + xn > n 2xn−1 .

Tập san Toán học STAR EDUCATION


Nhìn lại các bài toán hình phẳng
trong đề tuyển sinh chuyên Toán
PTNK

Nguyễn Phi Hùng


(GV trường PTNK TPHCM)

Trong đề thi trường Phổ thông Năng khiếu, bài toán hình học luôn là bài toán thú vị và
khó, trong đề thi toán chung bài hình học cũng khai thác các tính chất hình học quen
thuộc, nhưng luôn thay đổi về hình, có khi cho tam giác, có khi cho hình vuông, bình bình
hành,...đó là một điều khá thú vị. Trong các câu có sự phân loại rất rõ ràng, những bạn
học chắc mới kịp làm trọn vẹn bài hình trong thời gian phòng thi. Bài viết này nhìn lại
một số bài toán hình trong đề thi môn toán không chuyên để các em ôn tập tốt.

MỘT SỐ CÂU HỎI THƯỜNG GẶP

ˆ Tính độ dài, tính góc, tính tỉ số đoạn thẳng.

ˆ Chứng minh góc bằng nhau, đoạn thẳng bằng nhau.

ˆ Chứng minh tứ giác nội tiếp.

ˆ Chứng minh đường thẳng là tiếp tuyến của đường tròn.

Vì muốn các bạn vừa ôn tập vừa giải các bài toán, nên có trình bày thêm một số
vấn đề lưu ý khi ôn tập.

MỘT SỐ TRỌNG TÂM CẦN NẮM VỮNG

ˆ Ôn tập tính chất và dấu hiệu nhận biết các hình: hình thang, hình thang
cân, hình bình hành, thoi, chữ nhật, vuông.

ˆ Ôn tập công thức tính diện tích tam giác, diện tích các hình, diện tích
hình tròn.

ˆ Định lý Thalet và tam giác đồng dạng, tính chất đường phân giác.

ˆ Ôn tập các hệ thức lượng, tỉ số lượng giác, tính độ dài hoặc góc trong tam
giác vuông.

23
24 NGUYỄN PHI HÙNG

ˆ Ôn tập đường tròn, các tính chất cơ bản.

ˆ Tính chất và phương pháp chứng minh tiếp tuyến.

ˆ Xác định được các loại góc: góc nội tiếp, góc ở tâm, góc giữa tia tiếp tuyến
và dây cung. Mối liên hệ giữa các góc.

ˆ Các phương pháp chứng minh tứ giác nội tiếp.

ˆ Các bài toán phụ quen thuộc.

Lúc giải bài toán hình học, các bạn cần chú ý một số vấn đề sau:

MỘT SỐ CHÚ Ý KHI GIẢI BÀI TOÁN HÌNH HỌC

1. Vẽ hình sớm và đúng giả thiết.

ˆ Chú ý các giả thiết, tam giác nhọn, tam giác tù, vẽ chính xác số đo
góc.
ˆ Tên điểm đúng.
ˆ Không vẽ hình quá đặc biệt.
ˆ Kí hiệu các góc vuông, hai góc bằng nhau, đoạn thẳng bằng nhau lên
hình vẽ.

2. Không được áp dụng các định lí, bài toán ngoài SGK mà không chứng
minh.

3. Trình bày rõ ràng, logic, tính toán cẩn thận, độ dài.

Trước khi bước vào các đề thi ta cùng làm bài toán sau, đây là những tính chất rất
quen thuộc.

Ví dụ 1. (Các tính chất cơ bản về đường cao và trực tâm). Cho tam giác ABC
nội tiếp đường tròn tâm O. Các đường cao AD, BE, CF cắt nhau tại H. Vẽ đường
kính AP , gọi M là trung điểm BC.

1. Chứng minh các tứ giác AEHF, BF DC nội tiếp.

2. Chứng minh BHCP là hình bình hành và M là trung điểm P H.

3. Chứng minh AH = 2OM .

4. AD cắt (O) tại Q. Chứng minh BC là trung trực của HQ. Tứ giác BCP Q
là hình thang cân.

5. Cho ∠BAC = 60◦ . Chứng minh BHOC nội tiếp và ∠OHC = 30◦ .

6. Vẽ tiếp tuyến tại A là d. Chứng minh d||EF và OA⊥EF .

7. Vẽ QX⊥AB, QY ⊥AC. Chứng minh XY đi qua điểm D.

Tập san Toán học STAR EDUCATION


NGUYỄN PHI HÙNG

A
x
E
F
H

O Y

D
B C
M

Q P

Lời giải. 1. Tự chứng minh.


2. Ta có ∠ABP = 90◦ và ∠AF C = 90◦ , suy ra CH||BP ; chứng minh tương tự
thì BH||CP .
Do đó BHCP là hình bình hành.
M là trung điểm BC nên cũng là trung điểm HP .
3. Tam giác AHP thì OM là đường trung bình nên AH = 2OM .
4. Ta có ∠BHQ = ∠ACB = ∠AQB nên tam giác BHQ cân, do đó BC là trung
trực của HQ.
P Q⊥AQ và BC⊥AQ nên BC||P Q, mà CQ = CH = BP nên BCP Q là hình
thang cân.
5. BAC = 60◦ , suy ra BHC = 180◦ − ∠BAC = 120◦ và ∠BOC = 2∠BAC =
120◦ .
Do đó ∠BOC = ∠BHC, suy ra BHOC nội tiếp.
Khi đó ∠OHC = ∠OBC = 30◦ .
Tâm đường tròn là điểm chính giữa cung BC.
6. Ta có ∠xAB = ∠ACB = ∠AF E, suy ra Ax||EF .
Mà OA⊥Ax, suy ra OA⊥EF .
7. Các tứ giác QXBD, QDY C, AXQY nội tiếp.
Suy ra ∠XDQ = ∠QBX = ∠ACQ = ∠ADY , do đó X, Y, D thẳng hàng.

Ví dụ 2. (Mô hình 2 tiếp tuyến song song) Cho đường tròn tâm O đường kính
AB = 2R, tiếp tuyến tại A là d, tiếp tuyến tại B là d0 , C thuộc đường tròn. Tiếp

Tập san Toán học STAR EDUCATION


26 NGUYỄN PHI HÙNG

tuyến tại C cắt d, d0 lần lượt tại D, E.

1. Chứng minh ED = AD + BE.

2. Chứng minh ∠DOE = 90◦ và AD · BE = R2 .

3. BC cắt d tại F , chứng minh DA = DF .

4. Vẽ đường cao CH của tam giác ABC. Chứng minh BD đi qua trung điểm
của CH.

F
C

D M

A B
H O

Lời giải. 1. CD = DA, CE = EB (t/c tt cắt nhau), suy ra DE = AD + BE.


2. OD là phân giác ∠COD và OE là phân giác ∠BOC, mà hai góc ∠AOC, ∠BOC
kề bù nên OD⊥OE.
Ta có AD · BC = CD · CE = OC 2 = R2 .
3. OD⊥AC, suy ra OD||BF ; mà O là trung điểm AB nên D là trung điểm AF .
4. Gọi M là giao điểm của BD và CH.
CM BM MH
Ta có = = , và DF = DA suy ra M C = M D.
DF BD AD


Ví dụ 3. Cho tam giác ABC có ∠BAC = 75◦ , ∠BAC = 45◦ , AC = a 2. AK
vuông góc với BC và K thuộc BC.

1. Tính độ dài các đoạn KC và AB theo a.

Tập san Toán học STAR EDUCATION


NGUYỄN PHI HÙNG

2. Gọi H là trực tâm và O là tâm đường tròn ngoại tiếp tam giác ABC. Tính
∠OHC.

3. Đường tròn tâm I nội tiếp tam giác ABC. Tính bán kính đường tròn ngoại
tiếp tam giác HIO theo a.

I O
E H

A C

AC
Lời giải. 1. Tam giác ACK vuông cân tại C, suy ra AK = √ = a
√ 2
AK 3 2a
sin ∠ABK = = ⇒ AB = √
AB 2 3

2. Ta có ∠AOC = 2∠ABC = 1200 và ∠AHC = 2∠EHF = 1800 − ∠BAC =


1200 .
Suy ra ∠AHC = ∠AOC, suy ra AHOC nội tiếp.
Do đó ∠OHC = ∠OAC = 300

3. Ta có ∠AIC = 1800 − ∠IAC − ∠ICA = 1800 − 11 (∠BAC + ∠ACB) = 1200 =


∠AOC.
Do đó tứ giác AIOC nội tiếp.
Vậy 5 điểm A, H, I, O, C cùng thuộc đường tròn. Gọi D là điểm chính giữa
cung AC. Ta có OAD và OCD đều, suy ra DA = DC = √ DO, hay D là tâm
AB a 2
đường tròn ngoại tiếp, và bán kính DO = DA = √ = √ .
2 3

Tập san Toán học STAR EDUCATION


28 NGUYỄN PHI HÙNG

Ví dụ 4. (PTNK 2020-Tam giác cân tù) Tam giác ABC nội tiếp đường tròn (T )
có tâm O có AB = AC và ∠BAC > 90◦ . Gọi M là trung điểm của AC, tia M O
cắt (T ) tại D, BC lần lượt cắt AO và AD tại N , P .

1. Chứng minh OCM N là tứ giác nội tiếp và ∠BDC = 4∠ODC.

2. Phân giác góc ∠BDP cắt BC tại E, M E cắt AB tại F . Chứng minh
CA = CP và M E vuông góc với DB.
DE
3. Chứng minh tam giác M N E cân. Tính .
DF

B E C
P N
F

Lời giải. 1. ˆ Ta có: AB = AC, OB = OC


⇒ OA là đường trung trực của BC ⇒ OA⊥BC (tại N )
M là trung điểm của AC ⇒ OM ⊥AC (tại M )
Xét tứ giác ON M C có: ∠ON C = ∠OM C = 90◦ ⇒ ON M C nội tiếp.
ˆ Ta có: AB = AC ⇒ ∠BDA = ∠CDA
M D là trung trực AC ⇒ 4DAC cân tại D ⇒ DM là phân giác ∠ADC
1 1
⇒ ∠CDO = ∠CDA = ∠CDB hay ∠CDB = 4∠CDO.
2 4
2. ˆ Ta có: ∠ACB = ∠ADB = ∠ADC
4ADC cân tại D ⇒ ∠DAC = ∠ACD = ∠ACB + ∠DCB (1)
Và ∠AP C = ∠ADC + ∠DCB = ∠ACB + ∠DCB (2)
Từ (1) và (2) ⇒ ∠AP C = ∠DAC ⇒ 4ACP cân tại C ⇒ CA = CP
ˆ Gọi K là giao điểm của M E với BD
1 1
∠EDP = ∠BDA = ∠BDC
2 4
Tập san Toán học STAR EDUCATION
NGUYỄN PHI HÙNG

1
⇒ ∠EDM = ∠BDC = ∠ACP
2
⇒ DEM C nội tiếp ⇒ ∠DEC = ∠DM C = 90◦

∠BEK = ∠M EC = ∠CDM

∠DBE = ∠DAC = ∠DCA
⇒ ∠BEK + ∠DBE = ∠CDM + ∠DCA = 90◦ ⇒ ∠BKE = 90◦
⇒ M E vuông góc với DB.
(
∠CN M = ∠ABC = ∠ADC
3. ˆ Ta có: 1
∠N EM = ∠CDM = ∠ADC
2
Mà ∠CN M = ∠N EM + ∠N M E
⇒ ∠N M E = ∠N EM ⇒ 4N M E cân tại N .
ˆ ∠DEF = ∠ACD = ∠DBF
⇒ DF BE nội tiếp ⇒ ∠DF B = 90◦ ⇒ ∠DF E = ∠DBE = ∠DEF ⇒
DE
4DF E cân D ⇒ =1
DF

Ví dụ 5. (PTNK 2017 -Tam giác tù) Tam giác ABC nội tiếp đường tròn (T )
tâm O, bán kính R; ∠BAC = 120◦ , ∠ABC = 45◦ , H là trực tâm. AH, BH, CH
lần lượt cắt BC, CA, AB tại M , N , P .
MP
1. Tính AC theo R. Tính số đo góc ∠HP N và
MN
2. Dựng đường kính AD, HD cắt (T ) tại E (E 6= D) và cắt BC tại F . Chứng
minh các điểm A, N , H, P , E cùng thuộc một đường tròn và F là trung
điểm của HD.

3. Chứng minh AD⊥N P . Tia OF cắt (T ) tại I, chứng minh I là tâm đường
tròn ngoại tiếp tam giác HBC và AI đi qua trung điểm của M P

Lời giải. 1. Ta có ∠AOC = 2∠ABC = 90◦ (góc ở tâm bằng 2 lần góc nội tiếp
cùng chắn 1 cung).
2 2 2 2
√ ra tam giác OAC vuông tại O, suy ra AC = OA + OC = 2R ⇒ AC =
Suy
R 2.
Tứ giác BN P C có ∠BN C = ∠BP C = 90◦ nên là tứ giác nội tiếp.
Suy ra ∠HP N = ∠HBC = 90◦ − ∠ACB = 75◦ .
Các tứ giác AN BM và BN P C nội tiếp nên ∠AN M = ∠ABC = 45◦ , ∠CN P =
∠P BC = 45◦ .
Suy ra ∠M N P = ∠CN P + ∠CP N = 90◦ .
Và ∠N P B = ∠ACB = ∠AP M = 15◦ , suy ra ∠N P M = ∠N P B + ∠AP M =
30◦ .
MN 1 MP
Khi đó = sin ∠N P M = sin 30◦ = . Suy ra = 2.
MP 2 MN

Tập san Toán học STAR EDUCATION


30 NGUYỄN PHI HÙNG

2. Ta có ∠AEF = 90◦ (góc nội tiếp chắn nửa đường tròn).


Ta có ∠AN H = ∠AEH = ∠AP H = 90◦ nên 5 điểm A, N, H, P, E cùng thuộc
đường tròn đường kính AH.
Ta có ∠ABD = ∠ACD = 90◦ (góc nội tiếp chắn nửa đường tròn), suy ra
AB⊥BD, suy ra HC||BD.
Tương tự ta có HB⊥CN, CD⊥CN , suy ra HB||CD. Tứ giác HBDC có các
cặp cạnh đối song song nên là hình bình hành, suy ra F là trung điểm của
BC và HD.

3. Ta có ∠CAD = 45◦ = ∠CN M , suy ra AD||M N . Mà M N ⊥N P , suy ra


AD⊥N P .
Ta có OF là trung trực của BC, suy ra IB = IC. ∠BDC = 180◦ − ∠BAC =
60◦ .
1
Xét tam giác IOC có ∠IOC = ∠BOC = ∠60◦ . Suy ra tam giác IOC đều.
2
Do đó IB = IC = IO. (1)
Mặt khác tứ giác HBOC có ∠BHC + ∠BOC = 60◦ + 120◦ = 180◦ , suy ra
HBOC nội tiếp. (2)
Từ (1) và (2) suy ra I là tâm đường tròn ngoại tiếp tam giác HBC.
Tam giác P BC có ∠BP C = 90◦ , ∠P BC = 45◦ nên là tam giác vuông cân,
suy ra P B = P C, suy ra P thuộc trung trực của BC. Do đó P, O, I thẳng
hàng và P I⊥BC, suy ra P I||AM .
Mặt khác ta có ∠BIH = 2∠HCB = 90◦ , suy ra HBM I nội tiếp, suy ra
∠IM C = ∠BHI = 45◦ . Suy ra ∠IM C = ∠P BC = 45◦ , suy ra IM ||P A.
Tứ giác AP IM có 2 cặp cạnh đối song song nên là hình bình hành, suy ra AI
qua trung điểm của M P .

Sau đây là một số bài toán với mô hình là tứ giác, hình thoi, hình bình hành, hình
chữ nhật. Trong các bài toán này vẽ hình là kĩ năng quan trọng để giải bài.

Ví dụ 6. (PTNK 2018) Tứ giác ABCD nội tiếp đường tròn (T ) tâm O, bán
kính R; ∠CAD = 45◦ , AC vuông góc với BD và cắt BD tại I, AD > BC. Dựng
CK vuông góc với AD (K ∈ AD), CK cắt BD tại H và cắt (T ) tại E (E 6= C)

1. Tính số đo góc ∠COD. Chứng minh các điểm C, I, K, D cùng thuộc một
đường tròn và AC = BD.

2. Chứng minh A là tâm đường tròn ngoại tiếp tam giác BHE. Tính IK theo
R.

3. IK cắt AB tại F . Chứng minh O là trực tâm tam giác AIK và CK · CB =


CF · CD.

Tập san Toán học STAR EDUCATION


NGUYỄN PHI HÙNG

Lời giải. 1. ˆ Ta có COD


\ = 2CAD \ = 2 × 45◦ = 90◦ (góc ở tâm bằng hai lần
góc nội tiếp cùng chắn cung CD).
CID
[ = CKD \ = 90◦
Suy ra A, I, K, D cùng thuộc đường tròn đường kính CD.
ˆ 4IAD có Ib = 90◦ , IAD
[ = 45◦ suy ra IDA
[ = 45◦ , do đó 4IAD vuông
cân tại I.
Suy ra IA = ID (1).
ˆ CBI
[ = IAD [ = 45◦ , 4ICB có CIB
[ = 90◦ , CBI
[ = 45◦ suy ra ICB
[ =

45 , do đó 4ICB vuông cân tại I.
Từ đó suy ra IC = IB (2).
ˆ Từ (1) và (2) suy ra IA + IC = IB + ID, do đó AC = BD.

2. Tứ giác 4IHK có Ib + K b = 90◦ + 90◦ = 180◦ .


Suy ra AIHK nội tiếp.
Suy ra CHB
\ = CAD \ = 45◦ = CBH.\
Do đó 4CBH vuông cân tại C có CI là đường cao nên cũng là đường
trung trực đoạn thẳng BH.
Suy ra AB = AH. (3)

Ta có HAD
\ = HCD\ (cùng phụ ADC),
\
Mà HCD
\ = DAE\ nên HAD
\ = DAE.\
Suy ra 4AKH = 4AKE (g.c.g).
Suy ra AH = AE (4)

ˆ Từ (3), (4) ta được AH = AE = AB nên A là tâm đường tròn ngoại tiếp


4BHE.
ˆ 4AKC v 4AID suy ra AK.AD = AI.AC
AK AI
Do đó = .
AC AD
IK AK \ = cos 45◦ = √1 .
Suy ra 4AIK v 4ACD suy ra = = cos CAK
CD AC 2

Tập san Toán học STAR EDUCATION


32 NGUYỄN PHI HÙNG

√ √
Mà CD = CO2 + OD2 = R 2.
Do đó IK = R.

IA = ID
3. ˆ Ta có suy ra IO là trung trực AD, do đó IO ⊥ AD. (5)
 OA = OD
KC = KA
suy ra KO là trung trực AC, do đó KO ⊥ AC. (6)
OC = OA

ˆ Từ (5), (6) suy ra O là trực tâm 4AIK.


ˆ Ta có CAF
[ = CDB \ (cùng bằng nửa số đo cung CB).
CDB
\ = CKF \ (Tứ giác CIKD nội tiếp).
Suy ra CAF
[ = CKF\ , do đó tứ giác CKAF nội tiếp.
Từ đó suy ra CF
[ A = 180◦ − CKA
\ = 90◦ .
CB CF
ˆ Ta có 4CBF v 4CDK(g.g). Do đó = . Suy ra CB.CK =
CD CK
CD.CF .

Ví dụ 7. (PTNK 2013) Cho tứ giác ABCD nột tiếp đường tròn đường kính AC,
AC = 2a. Gọi M ,N lần lượt là trung điểm của AB và AD, tam giác ABD đều.

1. Tính BC và CN theo a.

2. Gọi H là trực tâm của tam giác CM N , M H cắt CN tại E, M N cắt AC


tại K. Chứng minh năm điểm B, M , K, E, C cùng thuộc một đường tròn
(T ). Đường tròn (T ) cắt BD tại F (F 6= B), tính DF theo a.

3. KF cắt M E tại I. Chứng minh KM tiếp xúc với đường tròn ngoại tiếp tam
giác M IF . Tính góc IN D.

HO P
A K C
I
E
N F

Lời giải. 1. Ta có OB = OD, AB = AD nên AO là trung trực của BD.


∠BOC = 2∠BAC = 600 nên tam giác OBC đều, suy ra BC = OC = a.
√ √ 1
AD = AC 2 − CD2 = a 3 (vì BC = CD = OC = a), suy ra DN = AD =
2

Tập san Toán học STAR EDUCATION


NGUYỄN PHI HÙNG


a 3
, suy ra
2 √

r
3 a 7
CN = CD2 + DN 2 = a2 a2 =
4 2

2. Ta có M N//BD, suy ra M N ⊥AC, suy ra H thuộc AC.


Ta có ∠CBM = ∠CEM = ∠CKM = 900 nên 5 điểm B, C, M , K, E cùng
thuộc đường tròn.
Ta có ∠KF B = ∠KCB = ∠ADB, suy ra KF//AD.
Gọi P là giao điểm của AC và BD. Tam giác P AD có KF//AD mà K √ là
1 a 3
trung điểm của AP suy ra F là trung điểm P D. Suy ra F D = P D = .
2 4
3. Ta có ∠KM I = ∠KCE, ∠KCM = ∠KF M và ∠KCE = ∠KCM vì tam
giác CM N cân.
Do đó ∠KM I = ∠KF M . (1)
Vẽ tia tiếp tuyến M x của đường tròn ngoại tiếp tam giác M F I.
Ta có ∠xM I = ∠IF M (2)
Từ (1) và (2) suy ra ∠KM I = ∠xM I, suy ra M x và M K trùng nhau. Hay
M K là tiếp tuyến của đường tròn ngoại tiếp tam giác M F I.
Ta có 4KM I v 4KF M , suy ra KI.KF = KM 2 = KN 2 , suy ra 4KIN v
4KN F , suy ra ∠KIN = ∠KN F = 900 , mà KF//N D, suy ra ∠IN D = 900 .

Ví dụ 8. (PTNK 2014 - Hình vuông) Cho hình vuông ABCD có AB = 2a, AC


cắt BD tại I. Gọi T là đường tròn ngoại tiếp tam giác CID, BE tiếp xúc với T
tại E (E khác C). DE cắt AB tại F .

1. Chứng minh tam giác ABE cân. Tính AF theo a.

2. BE cắt AD tại P . Chứng minh đường tròn ngoại tiếp tam giác ABP tiếp
AP
xúc với CD. Tính
PD
3. AE cắt T tại M (M khác E). Tính AM theo a.

Lời giải. 1. Ta có ∠BEC = ∠ADC = 600 (ADCE nội tiếp) và ∠ABC =


∠ADC = 600 (ABCD là hình bình hành), tam giác BCE có ∠EBC =
∠BEC = 600 nên là tam giác đều.
Ta có ∠DCE = 1800 −∠DAE = 60◦ , suy ra ∠DCE = ∠ADC nên hình thang
AECD là hình thang cân. Khi đó ∠ACD = ∠EDC, tam giác ICD cân tại I.
Ta có IC = ID, OC = OD nên OI là trung trực của CD. Do đó OI⊥CD

2. Ta có K là trung điểm BD nên K cũng là trung điểm AC do ABCD là hình


bình hành. Khi đó OK⊥AC và OK là trung trực của AC. Suy raM A = M C.
Suy ra ∠M AC = ∠ACM .

Tập san Toán học STAR EDUCATION


34 NGUYỄN PHI HÙNG

Mà ∠ACM = ∠IDM
Từ đó ∠IDM = ∠M AC. Suy ra tứ giác AIM D nội tiếp.

3. Ta có JK⊥AC. Suy ra I, K, O thẳng hàng. Do tam giác ABC và tam giác


ACD bằng nhau nên JK = OK.
1
Mặt khác ∠KJC = ∠AJC = ∠ABC = 600
2
KJ 1
Khi đó = cot ∠KJC = √
CK 3
1
Mà OJ = 2JK, DE = AC (AECD là hình thang cân), OK = AC.
2
OJ 2JK 2JK KJ 1 OJ 2
Khi đó = = = = √ . Vậy =√
DE AC 2CK CK 3 DE 3

Ví dụ 9. (PTNK 2019-Hình chữ nhật và tam giác vuông) Hình chữ nhật ABCD
nội tiếp đường tròn (T ) có tâm O, bán kính R = 2a. Tiếp tuyến của (T ) tại C
cắt các tia AB, AD lần lượt tại E, F .

1. Chứng minh AB · AE = AD · AF và BEF D là tứ giác nội tiếp.

2. Đường thẳng d đi qua A, d vuông góc với BD và d cắt (T ), EF theo thứ tự


tại M , N (M 6= A). Chứng minh BM N E là tứ giác nội tiếp và N là trung
điểm của EF .

3. Gọi I là tâm đường tròn ngoại tiếp tam giác BEF . Tính IN theo a

Tập san Toán học STAR EDUCATION


NGUYỄN PHI HÙNG

Lời giải. 1. ABCD là hình chữ nhật nội tiếp đường tròn (O) nên O là trung
điểm của AC, BD. Ta có OC⊥EF (do EF là tiếp tuyến), AD⊥CD do ABCD
là hình chữ nhật.
Tam giác ACF vuông tại C có CD là đường cao nên AD · AF = AC 2 .
Tương tự, tam giác ACE vuông tại C có CB là đường cao nên AB·AE = AC 2 .
Do đó AD · AF = AB · AE.
AD AB
Suy ra = .
AE AF
AD AB
Xét tam giác ABD và AF E có ∠A chung và = nên 4ABD v
AE AF
4AF E, suy ra ∠ABD = ∠AF E, suy ra tứ giác BDF E nội tiếp.
(Cách khác: ∠ABD = ∠ACD mà ∠ACD = ∠AF E (cùng phụ ∠DCF ), suy
ra ∠ABD = ∠AF E).

2. Ta có ∠AM B = ∠ACB (cùng chắn cung AB), mà ∠ACB = ∠BEN (cùng


phụ ∠BCE). Suy ra ∠AM B = ∠BEN , suy ra BEN M nội tiếp. Ta có
∠BM A = ∠BDA (cùng chắn cung AB), mà ∠BDA = ∠BAM (cùng phụ
với ∠ABD)
Suy ra ∠BM A = ∠BAM = ∠N AE. Vậy ∠N EA = ∠N AE. Tam giác N AE
có ∠N EA = ∠N AE nên cân tại N hay N A = N E.
Mà ∠N EA + ∠N F A = 90◦ = ∠N AE + ∠N AF , suy ra ∠N F A = ∠N AF ,
suy ra N A = N F . Vậy N E = N A = N F hay N là trung điểm EF .

3. Ta có N là trung điểm EF nên IN ⊥EF , mà AO⊥EF , suy ra IN k AO; và


IO⊥BD, AN ⊥BD, suy ra IO k AN ; Do đó tứ giác AN IO là hình bình hành,
suy ra IN = AO = R.

Ví dụ 10. (PTNK 2015 - Hình bình hành) Hình bình hành ABCD có ∠ADC =
600 và tam giác ACD nhọn. Đường tròn tâm O ngoại tiếp tam giác ACD cắt
cạnh AB tại E (E 6= A), AC cắt DE tại I.

1. Chứng minh tam giác BCE đều và OI⊥CD.

2. Gọi K là trung điểm BD, KO cắt DC tại M . Chứng minh A, D, M , I


cùng thuộc một đường tròn.
OJ
3. Gọi J là tâm đường tròn ngoại tiếp tam giác ABC. Tính .
DE


Lời giải. 1. Ta có ∠BEC = ∠ADC = 600 (ADCE nội tiếp) và ∠ABC =


∠ADC = 600 (ABCD là hình bình hành), tam giác BCE có ∠EBC =
∠BEC = 600 nên là tam giác đều.
Ta có ∠DCE = 1800 −∠DAE = 60◦ , suy ra ∠DCE = ∠ADC nên hình thang
AECD là hình thang cân. Khi đó ∠ACD = ∠EDC, tam giác ICD cân tại I.
Ta có IC = ID, OC = OD nên OI là trung trực của CD. Do đó OI⊥CD

Tập san Toán học STAR EDUCATION


36 NGUYỄN PHI HÙNG

2. Ta có K là trung điểm BD nên K cũng là trung điểm AC do ABCD là hình


bình hành. Khi đó OK⊥AC và OK là trung trực của AC. Suy raM A = M C.
Suy ra ∠M AC = ∠ACM .
Mà ∠ACM = ∠IDM
Từ đó ∠IDM = ∠M AC. Suy ra tứ giác AIM D nội tiếp.

3. Ta có JK⊥AC. Suy ra I, K, O thẳng hàng. Do tam giác ABC và tam giác


ACD bằng nhau nên JK = OK.
1
Mặt khác ∠KJC = ∠AJC = ∠ABC = 600
2
KJ 1
Khi đó = cot ∠KJC = √
CK 3
1
Mà OJ = 2JK, DE = AC (AECD là hình thang cân), OK = AC.
2
OJ 2JK 2JK KJ 1
Khi đó = = = =√
DE AC 2CK CK 3
OJ 2
Vậy =√
DE 3

1. Bài tập rèn luyện

Ví dụ 11. Cho tam giác ABC vuông cân tại C, CA = a. O là trung điểm AB.
M là tâm của đường tròn ngoại tiếp tam giác ACO. Từ B vẽ tiếp tuyến BD đến
(M ) (D khác C). CD cắt M B tại điểm H; BM cắt OC tại F .

1. Tính CD và diện tích tam giác BCD.

2. Tính tang của góc ∠OF B và chứng minh tứ giác ODM F nội tiếp.

3. OH cắt (M ) tại E. Tính DE và chứng minh tứ giác OM EB nội tiếp.

Ví dụ 12. Cho tam giác ABC nhọn nội tiếp đường tròn tâm O, bán kính R,
∠ACB = 45◦ . Các đường cao AD, BE cắt nhau tại H.

1. Chứng minh 5 điểm A, B, D, O, E cùng thuộc một đường tròn. Tìm tâm
M và tính bán kính đường tròn.

2. Gọi N là trung điểm CH. Tứ giác M DN E là hình gì? Tính diện tích tứ
giác.

3. N A cắt đường tròn đường kính AB tại P , CH cắt DE tại K. Chứng minh
B, K, P thẳng hàng.

Tập san Toán học STAR EDUCATION


NGUYỄN PHI HÙNG

Ví dụ 13. (PTNK 2016) Tam giác ABC đều có tâm O,AB = 6a và các điểm
M, N lần lượt thuộc các cạnh AB, AC mà AM = AN = 2a. Gọi I, J, K lần lượt
là trung điểm của BC, AC và M N .

1. Chứng minh các điểm M, N, B, C cùng thuộc một đường tròn T. Tính diện
tích tứ giác BM N C theo a.

2. Tính bán kính đường tròn ngoại tiếp tam giác IJK. Chứng minh đường tròn
đường kính N C tiếp xúc với AI.

3. AE tiếp xúc với đường tròn T tại E (E, B cùng phía đối với AI).Gọi F là
trung điểm OE, tính số đo ∠AF J.

Ví dụ 14. Cho hình thang ABCD có AB = a, AD = CD = 2a, ∠A = ∠D = 90◦ .


Đường tròn tâm O ngoại tiếp tam giác ABC. Tiếp tuyến tại B của (O) cắt AC
tại E, OE cắt cạnh AD tại F .

1. Tính ∠BOC và bán kính đường tròn (O) theo a.

2. Chứng minh EBCO là hình bình hành và OA tiếp xúc với đường tròn ngoại
tiếp tam giác AEF .

3. BF cắt (O) tại K. Tính ∠AKD và chứng minh KA qua trung điểm EB.

Ví dụ 15. Cho hình bình hành ABCD có ∠BAD = 60◦ , ∠ADB = 45◦ , AB = 2a.
Đường tròn tâm O ngoại tiếp tam giác ABD cắt BC tại E, AE cắt BD tại G.

1. Chứng minh OBCD nội tiếp. Tính ∠COD. Tính OD theo a.

2. Gọi K là điểm đối xứng của E qua G. BK cắt AD tại L. Chứng minh
A, B, G, L cùng thuộc đường tròn và DK⊥AB.

3. OC cắt BD tại F . Chứng minh OF GK nội tiếp và tính OK · OC.

Ví dụ 16. Cho hình thoi ABCD có AB = AC = 2a. Đường tròn tâm O ngoại
tiếp tam giác ABC cắt BD tại E (khác B). AE cắt CD tại M .

1. Chứng minh DA, DC là tiếp tuyến của (O) và CM theo a.

2. Từ M vẽ tiếp tuyến M F đến (O), F khác C. DF cắt (O) tại K. Chứng


minh C, O, K thẳng hàng và tính DF .

Tập san Toán học STAR EDUCATION


38 NGUYỄN PHI HÙNG

3. Cho EF cắt AD tại G và CF cắt AE tại H. Chứng minh AGF H nội tiếp
và GH⊥BD.

Ví dụ 17. Cho đường tròn tâm O bán kính R, điểm P nằm ngoài đường tròn sao
cho OP = 2R. Từ P vẽ các tiếp tuyến P A, P B với A, B là các tiếp điểm. Gọi H
là giao điểm của OP và AB. Vẽ đường kính AC, CP cắt (O) tại điểm D khác
C.

1. Chứng minh tứ giác OHDC nội tiếp và tính HD theo R.

2. DH cắt (O) tại E. Chứng minh AECB là hình chữ nhật và tính diện tích
tứ giác theo R.

3. AB cắt CP tại Q, OP cắt AD tại K. Gọi L là trung điểm của AP , KL


cắt OQ tại J. Chứng minh QK k AC và AJDL nội tiếp.

Tập san Toán học STAR EDUCATION


Các bài toán tổ hợp từ đề JBMO
(tiếp theo)

Lê Phúc Lữ
(GV Đại học KHTN TPHCM)
Bùi Khánh Vĩnh
(SV Đại học Bách Khoa TPHCM)

Tiếp theo kỳ trước, trong tập san lần này, nhóm tác giả xin giới thiệu một số bài toán
tổ hợp trong kỳ thi Olympic thiếu niên Balkan (JBMO) từ 1997 đến nay, tập trung vào
phần hình học tổ hợp. Nếu các bạn độc giả có gộp các bài toán của số tập san lần trước
và lần này lại thì sẽ thấy một điều rằng trong 24 diễn ra kỳ thi, chỉ có riêng 2002 và 2018
thì đề thi không có cho tổ hợp.

Bài 1. (JBMO 1997) Trong một hình vuông đơn vị, có 9 điểm phân biệt mà
không có ba điểm nào thẳng hàng. Chứng minh rằng có ba đỉnh tạo thành một tam
giác có diện tích không lớn hơn 81 .

1
Lời giải. Chia hình vuông đã cho thành 4 hình vuông nhỏ cùng kích thước là 4
thì
theo nguyên lý chuồng bồ câu, phải có ba điểm cùng thuộc một hình vuông.
Ta sẽ chứng minh bổ đề quen thuộc sau: Ba điểm nằm trong hình chữ nhật thì diện
tích tam giác tạo thành luôn không vượt quá nửa diện tích hình chữ nhật. Trước
hết, ta thấy rằng chỉ cần xét trường hợp các điểm nằm trên các cạnh của hình chữ
nhật, vì nếu không, ta xét một điểm nằm trong hình tam giác và vẽ các tia cắt các
cạnh của hình chữ nhật, tạo thành tam giác lớn hơn chứa tam giác ban đầu.

Có hai trường hợp cần xét:


1. Nếu cả ba đỉnh đều thuộc các cạnh khác nhau, giả sử M ∈ AB, N ∈ BC, P ∈
CD như hình bên trái. Khi đó, vẽ đường qua N song song với AB, CD cắt
AD ở Q. Rõ ràng SM N P ≤ SM N P Q = 12 SABCD .

39
40 LÊ PHÚC LỮ - BÙI KHÁNH VĨNH

2. Nếu có hai đỉnh nào đó chung cạnh, giả sử ta có hình bên phải thì SM N P ≤
SM AD = 12 SABCD .

Do đó, bổ đề được chứng minh. Áp dụng trực tiếp bổ đề trên, ta có đpcm.

Bài 2. (JBMO 1999) Gọi S là hình vuông có cạnh 20 và M là tập hợp bốn đỉnh
của hình vuông cùng 1999 điểm nữa nằm bên trong hình vuông. Chứng minh rằng
1
có một tam giác có diện tích không vượt quá 10 và các đỉnh thuộc M.

Lời giải. Với một điểm, rõ ràng ta thu được 4 tam giác.

Nếu thêm vào một điểm nữa, ta có hai trường hợp:

ˆ Nếu điểm mới thuộc một trong các miền đang xét, số miền thu được sẽ là
4 − 1 + 3 = 6.

ˆ Nếu điểm mới nằm trên một trong các cạnh của các miền đang xét, số miền
thu được sẽ là 4 − 2 + 4 = 6.

Do đó, số tam giác luôn tăng lên 2. Một cách tổng quát, nếu có thêm n điểm nữa (sau
điểm đầu tiên) thì số miền là 4 + 2n. Với đề bài cho, số miền sẽ là 4 + 2 · 1998 = 4000
miền. Diện tích của hình vuông là 202 = 400 nên có một miền nào đó có diện tích
400 1
không quá 4000 = 10 , ta có đpcm.

Bài 3. (JBMO 2001) Một đa giác lồi có 1415 đỉnh và chu vi là 2001. Chứng
minh rằng có ba đỉnh của đa giác tạo thành tam giác có diện tích nhỏ hơn 1.

Lời giải. Gọi đa giác lồi đã cho là A1 A2 A3 . . . A1415 và giả sử phản chứng rằng cứ ba
đỉnh tùy ý của đa giác luôn tạo thành tam giác có diện tích không nhỏ hơn 1. Xét
riêng các tam giác có ba đỉnh liên tiếp là A1 A2 A3 , A2 A3 A4 , . . . , A1415 A1 A2 . Ta có
1 A2 A1 · A2 A3
SA1 A2 A3 = A2 A1 · A2 A3 · sin ∠A1 A2 A3 ≤ .
2 2
Mặt khác, theo bất đẳng thức AM-GM thì
p √
A2 A1 + A2 A3 ≥ 2 A2 A1 · A2 A3 ≥ 2 2.

Tập san Toán học STAR EDUCATION


LÊ PHÚC LỮ - BÙI KHÁNH VĨNH

Một cách tương tự, ta có thể đánh giá cho các tổng hai cặp cạnh liên tiếp khác.
Tiến hành cộng tất cả các đánh giá lại thì mỗi cạnh xuất hiện hai lần nên

1 √ √
A1 A2 + A2 A3 + · · · + A1415 ≥
· 1415 · 2 2 = 1415 2.
2

Tuy nhiên, dễ dàng kiểm tra được 2001 > 1415 2 nên đánh giá cuối ở trên dẫn đến
mâu thuẫn. Vậy phải có một tam giác có diện tích nhỏ hơn 1.

Bài 4. (JBMO 2003) Cho số nguyên dương n và xét n điểm trên mặt phẳng sao
cho không có ba điểm nào thẳng hàng. Biết rằng với mọi cách gọi tên các điểm
là A1 , A2 , A3 , . . . , An thì đường gấp khúc A1 A2 A3 . . . An không tự cắt. Tìm giá trị
lớn nhất của n.

Lời giải. Ta sẽ chứng minh rằng n ≥ 5 không thỏa mãn điều kiện đề bài. Xét 5
điểm trên mặt phẳng, ta sẽ chứng minh luôn tìm được một tứ giác lồi.
Thật vậy, đặt tên các điểm là A, B, C, D, E và xét bao lồi của hệ điểm này (là đa
giác lồi có số điểm ít nhất mà chứa toàn bộ các điểm của hệ điểm bên trong nó).
Nếu bao lồi đó là tứ giác hoặc ngũ giác thì nhận xét trên là đúng.
Nếu bao lồi là tam giác, khi đó bên trong nó sẽ có hai điểm. Giả sử D, E nằm trong
tam giác ABC là bao lồi. Đường thẳng DE không đi qua đỉnh nào nên phải cắt hai
cạnh của tam giác, giả sử là AB, AC và ta được B, C, D, E là tứ giác lồi.

Nhận xét được chứng minh.


Nhưng nếu có một tứ giác lồi thì ta gọi tên các đỉnh để tạo thành đường gấp khúc
đi qua hai đường chéo thì sẽ có đường gấp khúc tự cắt, không thỏa mãn đề bài. Do
đó n ≥ 5 không thỏa. Với n = 4, xét mô hình tứ giác lõm sau đây

Tập san Toán học STAR EDUCATION


42 LÊ PHÚC LỮ - BÙI KHÁNH VĨNH

Rõ ràng với mọi cách đặt tên thì đường gấp khúc A1 A2 A3 A4 sẽ không tự cắt. Vậy
giá trị lớn nhất của n là 4.

Bài 5. (JBMO 2004) Xét đa giác lồi P có n ≥ 4 đỉnh. Người ta chia đa giác
thành các tam giác con rời nhau, mỗi tam giác có đỉnh nằm trong n đỉnh của P .
Tô đen cho các tam giác có hai cạnh là cạnh của P, tô đỏ cho các tam giác có một
cạnh là cạnh của P và tô trắng cho các tam giác không có cạnh nào là cạnh của
P. Chứng minh rằng trong mọi cách chia thì số tam giác trắng nhiều hơn số tam
giác đen là 2.

Lời giải. Gọi b, r, w lần lượt là số tam giác đen, đỏ và trắng. Bằng quy nạp, ta có
thể chứng minh được rằng số lượng tam giác thu được luôn là n − 2 (ở trường hợp
n = 4, ta có hai tam giác, và cứ mỗi lần thêm một đỉnh, ta có thêm một tam giác).
Suy ra
b + r + w = n − 2.
Ngoài ra, mỗi cạnh của tam giác đều là cạnh của đúng một tam giác được chia ra,
do đó
2b + r = n.
Từ hai điều trên, ta có được w − b = 2. Đây chính là điều cần chứng minh.

Bài 6. (JBMO 2005) Chứng minh rằng tồn tại:

1. 5 điểm trên mặt phẳng sao cho trong tất cả các tam giác tạo thành từ các
điểm này, có đúng 8 tam giác vuông.

2. 64 điểm trên mặt phẳng sao cho trong tất cả các tam giác tạo thành từ các
điểm này, có ít nhất 2005 tam giác vuông.

Lời giải.
1. Ta chọn 4 đỉnh của hình vuông và tâm, tạo thành tất cả 5 điểm và 8 tam giác
vuông.

2. Xét lưới điểm có kích thước 8 × 8 thì lưới này có tất cả 64 điểm. Với mỗi điểm
thuộc lưới, rõ ràng có 7 điểm cùng hàng với nó và 7 điểm cùng cột với nó,
chúng tạo thành 7 · 7 = 49 tam giác vuông có đỉnh góc vuông là M.
Do đó, với 64 điểm, ta có ít nhất 64 · 49 = 3136 > 2005 tam giác vuông (rõ
ràng các tam giác này đều phân biệt vì các đỉnh góc vuông và hai cạnh góc
vuông tương ứng đều khác nhau).

Tập san Toán học STAR EDUCATION


LÊ PHÚC LỮ - BÙI KHÁNH VĨNH

Bài 7. (JBMO 2007) Cho 50 điểm trên mặt phẳng mà không có ba điểm nào
thẳng hàng. Mỗi điểm trong số các điểm này được tô bởi một trong bốn màu: xanh,
đỏ, tím, vàng. Chứng minh rằng có một màu được tô cho cả ba đỉnh của ít nhất
130 tam giác không cân.

Lời giải. Do 50 = 4 · 12 + 2 nên theo nguyên lý chuồng bồ câu, phải có ít nhất


13 điểm được tô cùng màu. Các điểm này sẽ tạo thành 13·12
6
= 78 đoạn thẳng và
13·12·11
6
= 286 tam giác.
Rõ ràng với mỗi đoạn AB, ta sẽ có tối đa hai tam giác cân được có cạnh đáy là AB
vì nếu không, chẳng hạn ta có ABC, ABD, ABE cân thì ba đỉnh C, D, E sẽ thẳng
hàng, mâu thuẫn.
Do đó, có tối đa 78 · 2 = 156 tam giác cân trong số các tam giác đã cho và tương
ứng, sẽ có ít nhất 286 − 156 = 130 tam giác không cân. Ta có đpcm.
Tổng quát kết quả ở bài trên, ta có: Với n > 8 điểm nằm trên mặt phẳng và không
có ba điểm nào thẳng hàng; ta sẽ có ít nhất

n(n − 1)(n − 8)
6
tam giác không cân tạo thành.

Bài 8. (JBMO 2009) Mỗi điểm trong số 2009 điểm cho trước trên mặt phẳng
được tô bởi một trong hai màu: xanh hoặc đỏ. Biết rằng mỗi đường tròn đơn vị có
tâm màu xanh đều đi qua đúng hai điểm đỏ. Hỏi có nhiều nhất bao nhiêu điểm
xanh?

Lời giải. Rõ ràng mỗi cặp điểm đỏ thuộc tối đa hai đường tròn có tâm màu xanh.
Gọi n là số điểm đỏ thì số cặp tương ứng sẽ là n(n−1)
2
và sẽ có không quá n(n − 1)
đường tròn có tâm xanh. Suy ra tổng số điểm không vượt quá

n(n − 1) + n = n2

điểm. Do đó n2 ≥ 2009 hay n ≥ 45. Khi đó, số điểm xanh không vượt quá 2009−45 =
1964 điểm.
Ta sẽ chỉ ra cách xây dựng thỏa đề bài. Xếp 45 điểm đỏ phân biệt lên một đoạn
thẳng có độ dài là 1. Sau đó vẽ 45 đường tròn đơn vị có tâm đỏ. Rõ ràng chúng sẽ đôi
một cắt nhau và các giao điểm đều phân biệt. Tổng số giao điểm là 45 · 44 = 1980.
Ta sẽ tô màu xanh cho đúng 1964 giao điểm trong số đó thì rõ ràng mỗi đường tròn
đơn vị có tâm là điểm xanh sẽ đi qua đúng hai điểm đỏ. Do đó, mô hình này thỏa
mãn đề bài.
Vậy số điểm xanh lớn nhất là 1964.

Tập san Toán học STAR EDUCATION


44 LÊ PHÚC LỮ - BÙI KHÁNH VĨNH

Bài 9. (JBMO 2001) Cho số nguyên dương n > 3. Tam giác đều ABC được chia
thành n2 tam giác con đồng dạng với nó (mỗi cạnh của tam giác con song song
với một trong các đoạn AB, BC, CA). Gọi m là số hình thoi chứa đúng hai tam
giác con và d là số hình thoi chứa đúng tám tam giác con. Tính giá trị m − d theo
n.

Lời giải. Ta sẽ đếm số hình thoi có đường chéo dài nhất vuông góc với cạnh BC
của tam giác rồi lấy số lượng đó nhân 3 là thu được m.
Xét các đường nằm ngang, song song với BC thì rõ ràng: đường thẳng cao nhất cho
ta đúng một hình thoi như đã nêu ở trên; đường tiếp theo cho ta hai hình thoi và
cứ thế, . . . đường cuối cùng cho ta n − 1 hình thoi. Suy ra
3n(n − 1)
m = 3(1 + 2 + 3 + · · · + n − 1) = .
2
Để đếm d, ta thực hiện tương tự nhưng số lượng hình thoi sẽ thay đổi từ 1 → n − 3
(đường thẳng cao thứ nhì cho đến đường thẳng thấp thứ nhì). Suy ra
3(n − 3)(n − 2)
d = 3(1 + 2 + 3 + · · · + n − 3) = .
2
Do đó, m − d = 6n − 9.

Bài 10. (JBMO 2012) Trên bảng, có n điểm đôi một nối với nhau bởi các đoạn
thẳng. Mỗi đoạn thẳng được tô bởi một trong n màu phân biệt. Với mỗi ba màu
tùy ý, tồn tại ba điểm mà ba đoạn nối chúng được tô tương ứng bởi ba màu đó.
Hỏi giá trị của n có thể là

1. n = 6?

2. n = 7 hay không?

Lời giải.
1. Câu trả lời là phủ định. Giả sử ngược lại rằng có thể có thể tô màu thỏa mãn
đề bài. Xét một màu trong số các màu, giả sử là xanh. Mỗi cạnh xanh là cạnh
của 4 tam giác có đỉnh trong số các điểm đã cho. Ngoài ra, có tất cả 5·4
2
= 10
cặp hai màu khác màu xanh và ứng với mỗi cặp như thế, cùng với màu xanh
thì luôn có một tam giác nhận chúng làm ba cạnh.

Suy ra phải có ít nhất 3 cạnh màu xanh, vì nếu không thì số tam giác nhận
cạnh xanh là cạnh của nó sẽ không quá 2 · 4 = 8, ít hơn số lượng ở trên. Rõ
ràng điều này cũng đúng với một màu tùy ý nên suy ra tổng số cạnh tạo thành
ít nhất là 6 · 3 = 18, trong khi chỉ có 6·5
2
= 15 cạnh mà thôi. Điều mâu thuẫn
này cho thấy không thể có câu trả lời ứng với n = 6.
2. Câu trả lời là khẳng định. Ta xét mô hình bên dưới với các điểm tạo thành
đa giác đều và các cạnh, đường chéo song song với nhau sẽ được đánh cùng
một số. Khi đó rõ ràng mỗi màu trong số các màu sẽ được dùng đúng 3 lần.
Do tính đối xứng nên ta sẽ chỉ cần xét các bộ ba các màu và có chứa màu 1.

Tập san Toán học STAR EDUCATION


LÊ PHÚC LỮ - BÙI KHÁNH VĨNH

Đó là các bộ:

(1, 2, 3) → (CEG); (1, 2, 4) → (ABF ); (1, 2, 5) → (ABG);


(1, 2, 6) → (ADG); (1, 2, 7) → (BDF ); (1, 3, 4) → (BCE);
(1, 3, 5) → (BCF ); (1, 3, 6) → (ADG); (1, 3, 7) → (AEG);
(1, 4, 5) → (CDE); (1, 4, 6) → (BEF ); (1, 4, 7) → (F GA);
(1, 5, 6) → (CEF ); (1, 5, 7) → (BF G); (1, 6, 7) → (ACE).

Do tính bình đẳng nên các bộ khác không chưa 1 vẫn tồn tại và vì thế nên mô
hình trên thỏa mãn điều kiện của đề bài.

Dưới đây là bài toán để bạn đọc tự luyện thêm.

Bài 11. (JBMO 2017) Xét đa giác đều P có 2n đỉnh là A1 A2 ...A2n với n > 1
nguyên dương. Ta gọi một điểm E nằm ngoài P là ‘nhìn thấy’ điểm S trên cạnh
nào đó của P nếu như đoạn thẳng SE cắt một cạnh của P tại vị trí khác S. Ta
tô màu các cạnh của P bởi 3 màu (không tô các đỉnh của P) sao cho mỗi cạnh
tô đúng một màu và mỗi màu được dùng ít nhất một lần. Ngoài ra, mỗi điểm bên
ngoài P thì có ’nhìn thấy’ tối đa 2 điểm khác màu trên cạnh của P. Tìm số cách
tô đa giác P thỏa mãn điều kiện trên.

Tập san Toán học STAR EDUCATION


Đề thi thử Toán không chuyên
tuyển sinh 10 THPT

Tập thể GV chuyên Star Education THCS

STAR - EDUCATION
MÔN: TOÁN - KHỐI 9
TUYỂN SINH VÀO LỚP 10 THPT
Năm học: 2021 - 2022
Đề thi thử Toán KC Thời gian: 120 phút - Không kể thời gian giao đề.

*************

   
5 5
Bài 1. Cho các biểu thức H1 = √ − 1−x : √ −1
√1 + x   √ 1 −x2
1+x x √

2x + 1 x
và H2 = √ − √ · √ − x
x x−1 x+ x+1 1+ x

(a) Rút gọn biểu thức H1 và H2 .


(b) Xét dấu của tích H1 · H2 .

(x − 3)(x − 5)
Bài 2. (a) Giải phương trình: √ +x−3=0
2x + 1
 √ 
(x − 3y − 1) x − y − 1 − x − y + 1 =0
(b) Giải hệ phương trình: √ √
x − 1 + 2 − 3y = 2

x2 + (m − 4)x − 2m2 + m + 3
Bài 3. Cho phương trình: √ =0
x−1

(a) Giải phương trình khi m = 2


(b) Tìm m để phương trình có hai nghiệm phân biệt x1 , x2 thỏa mãn điều
kiện x21 + x22 − x2 = 7.

Bài 4. (a) Bốn bạn An, Bình, Châu, Duy theo thứ tự An Bình
đứng ở 4 góc của sân tennis hình chữ nhật
11m
như hình vẽ. Trên sân có một quả banh ten- 14m
nis, biết khoảng cách từ quả banh đến An,
Bình, Châu lần lượt là 11m, 14m và 16m. ?
16m
Tính khoảng cách từ Duy đến quả banh?
Duy Châu

46
NGUYỄN TẤN PHÁT

(b) Sau trận đá bóng cuối cùng của trường Phổ thông Năng khiếu, người ta
1 2
thấy rằng số điểm của đội 10 Tin và đội GV lần lượt chiếm và tổng
4 7
số điểm ghi được của các đội. Đội Star ghi được 15 điểm. Không có đội
nào trong số 7 đội còn lại (ngoài 3 đội nêu trên) ghi được nhiều hơn 2
điểm. Hỏi tổng số điểm của 7 đội còn lại ghi được là bao nhiêu?

Bài 5. Cho tam giác ABC đều cạnh a. Về phía ngoài tam
giác, dựng các tam giác ABD và ACE vuông cân D E
tại A.

(a) Chứng minh tứ giác BCED nội tiếp. Tính A


chu vi tứ giác BCED theo a.

(b) BE, CD cắt nhau tại I. Chứng minh tứ giác


BIAD nội tiếp, tính độ dài AI theo a.

(c) BC cắt đường tròn ngoại tiếp tứ giác BIAD


tại điểm F khác B. EF cắt AI tại K. Tính B C
KA
∠BDF và .
KI

Tập san Toán học STAR EDUCATION


48 NGUYỄN TẤN PHÁT

STAR - EDUCATION
MÔN: TOÁN - KHỐI 9
TUYỂN SINH VÀO LỚP 10 THPT
Năm học: 2021 - 2022
Đề thi thử lần 1 Thời gian: 120 phút - Không kể thời gian giao đề.

——————

LỜI GIẢI
——————
Bài 1. (a) (Học sinh thiếu điều kiện không trừ điểm)
ˆ Điều kiện
 xác định: −1 <x <1


5 5
H1 = √ − 1−x : √ −1
1√+ x √ √ 1 − x2
5 − 1 − x2 1+x· 1−x
= √ · √
√ 1+x 5 − 1 − x2
= 1−x 0,5 điểm
ˆ Điều kiện
 xác định: x ≥√0 và x  6= 1 √
1+x x √

2x + 1 x
H2 = √ − √ · √ − x
x x − 1√ x√+ x + 1 1+ x
2x + 1 − x( x − 1) √ √
= √ √ · (x − x + 1 − x)
( x − 1)(x√ + x + 1)
x+ x+1 √
= √ √ · ( x − 1)2
( x − 1)(x + x + 1)

= x−1 0,5 điểm
(b) Điều kiện xác định: 0 ≤ x < 1
√ √
Ta có: H1 · H2 = 1 − x · ( x − 1) 0,25 điểm
√ √
Vì 1 − x > 0 và x − 1 < 0 với mọi 0 ≤ x < 1 suy ra H1 · H2 < 0
Vậy H1 · H2 < 0 0,25 điểm
−1
Bài 2. (a) Điều kiện xác định: x > 0,25 điểm
2
(x − 3)(x − 5)
√ +x−3=0
2x + 1

⇔ (x − 3)(x − 5) + (x − 3) 2x + 1 = 0

x = 3 (nhận)
⇔ √ 0,25 điểm
2x + 1 = 5 − x (1)

 x≤5 √

x≤5
(1) ⇔ 2 ⇔ x = 6 + 2√3 (loại) 0,5 điểm
x − 12x + 24 = 0
x = 6 − 2 3 (nhận)

(Học sinh không loại nghiệm trừ 0,25 điểm)


 √
Vậy S = 3; 6 − 2 3 .
2
(b) Điều kiện xác định: x ≥ 1, y ≤ , x ≥ y − 1
3
(Học sinh thiếu điều kiện xác định trừ 0,25 điểm)

Tập san Toán học STAR EDUCATION


NGUYỄN TẤN PHÁT

 √ 
(x
√ − 3y − 1)
√ x − y − 1 − x − y + 1 = 0 (1)
x − 1 + 2 − 3y = 2 (2)

x = 3y + 1 √
Ta có: (1) ⇔
x − y − 1 − x − y + 1 = 0 (3)
ˆ Thay
√ x√= 3y + 1 vào (2) ta có:
3y + 2 − 3y = p 2
⇔p 3y + 2 − 3y + 2 6y − 9y 2 = 4
⇔ 6y − 9y 2 = 1
⇔ 9y 2 − 6y + 1 = 0
1
⇔ y = ⇒ x = 2 (so với điều kiện ta nhận)
3  
1
Suy ra (x; y) = 2; . 0,5 điểm
3

ˆ Đặt t = x − y + 1 (t ≥ 0)
t = −1 (loại)
(3) ⇔ t2 − t − 2 = 0 ⇔
t = 2 (nhận)
Với t = 2 ⇔ x − y + 1 = 4 ⇔ x = y + 3
Thay
√ x =√ y + 3 vào (2), ta có:
y + 2 + 2 − 3y = p 2
⇔p y + 2 + 2 − 3y + 2 −3y 2 − 4y + 4 = 4
⇔  −3y 2 − 4y + 4 = y
y≥0

4y 2 + 4y − 4 = 0
y≥0


 √
−1 − 5
 

⇔  y= 2√
(loại)
 y = −1 + 5 (nhận)

 

√2 √
−1 + 5 5+ 5
Với y = ⇒x= (so với điều kiện ta nhận)
2 √ 2 √ !
5 + 5 −1 + 5
Suy ra (x; y) = ; . 0,5 điểm
2 2
  √ √ !
1 5 + 5 −1 + 5
Vậy nghiệm của hệ phương trình là 2; , ;
3 2 2

x2 + (m − 4)x − 2m2 + m + 3
Bài 3. √ = 0 (4)
x−1
Điều kiện xác định: x ≥ 0 và x 6= 1 0,25 điểm
x2 − 2x − 3

x = 3 (nhận)
(a) Thay m = 2 vào (4), ta có: √ =0⇔ 0,25
x−1 x = −1 (loại)
điểm
Vậy S = {3}
x2 + (m − 4)x − 2m2 + m + 3
(b) √ = 0 (1)
x−1
⇔ x2 + (m − 4)x − 2m2 + m + 3 = 0 (2)

Tập san Toán học STAR EDUCATION


50 NGUYỄN TẤN PHÁT

(Học sinh không đưa (1) thành (2) trừ 0,25 điểm)
Phương trình (1) có hai nghiệm phân biệt khi và chỉ khi phương trình
(2) có hai nghiệm phân biệt, không âm và khác 1 khi và chỉ khi:
2
 
(3m − 2) > 0 3
−1 ≤ x ≤
  
∆ > 0
 
 m≤4 2
 
 

2
  
S≥0 3
 
⇔ −1 ≤ x ≤ ⇔ m 6=
 P ≥0  2  3

 2
1 + m − 4 − 2m2 + m + 3 6= 0



 m 6= 0



 m 6= 0
 m 6= 1
m 6= 1

0,25 điểm
(Học sinh không cần giải hệ điều kiện chỉ cần đưa về theo
tham số m)
Với ∆ = (3m − 2)2 , ta suy ra x = m + 1 hoặc x = 3 − 2m.

ˆ Với x1 = m + 1 và x2 = 3 − 2m, ta có:"


m = 0 (loại)
x21 + x22 − x2 = 7 ⇔ 5m2 − 8m = 0 ⇔ 8 0,25 điểm
m= (loại)
5
ˆ Với x1 = 3 − 2m và x2 = m + 1, ta có: "
m = 2 (loại)
x21 + x22 − x2 = 7 ⇔ 5m2 − 11m + 2 = 0 ⇔ 1 0,25
m= (nhận)
5
điểm
1
Vậy m = thì phương trình (4) có hai nghiệm phân biệt thỏa yêu cầu.
5
0,25 điểm

Bài 4. (a) Gọi I là vị trí trái banh tennis. Từ I kẻ các đường thẳng vuông góc với
AB và AD.
Gọi M , N , P , Q lần lượt là giao điểm của 2 đường thẳng đó với AB,
BC, CD, DA.
Khi đó tứ giác AM IQ, M BN I là các hình chữ nhật.
Ta có: IA2 = IQ2 + AQ2 , IB 2 = IN 2 + BN 2
ID2 = IQ2 + QD2 , IC 2 = IN 2 + CN 2 0,25 điểm
Suy ra IA2 − ID2 = AQ2 − QD2 và IB 2 − IC 2 = BN 2 − CN 2 0,25
điểm
Vì AQ = BN và DQ = CN
Suy ra IA2 − ID2 = IB 2 − IC 2 0,25 điểm
⇒ ID2 = IA2 + IC 2 − IB 2 = 181

⇒ ID = 181 ≈ 13, 5 (m) 0,25 điểm

Tập san Toán học STAR EDUCATION


NGUYỄN TẤN PHÁT

(b) Gọi x (điểm) là tổng điểm số điểm của các đội. (x > 0).
y (điểm) là tổng điểm của 7 đội còn lại. (y > 0)
1 2
Số điểm của đội 10 Tin và đội GV là x và x
4 7
13
Suy ra số điểm của 8 đội còn lại là x.
28
13
Ta có phương trình: 15 + y = x 0,5 điểm
28
Không có đội nào trong 7 đội còn lại ghi nhiều hơn 2 điểm nên 0 < y ≤ 14.
13
Tứ đó suy ra 15 < x ≤ 29 ⇔ 32 < x ≤ 62
28
Vì x, y là các số tự nhiên nên x sẽ chia hết cho 28.
Do đó suy ra x = 56 (điểm)
Vậy tổng điểm của 7 đội còn lại là 11 điểm. 0,5 điểm

Bài 5. (a) ˆ 4ADB và 4AEC vuông cân tại A


suy ra AB = AC = AE = AD
Suy ra tứ giác BCED nội tiếp đường tròn tâm A bán kính AB. 0,5
điểm
√ √
ˆ Ta có: BD = CE = AB 2 + AD2 = a 2
Kẻ AT ⊥DE tại T suy ra T là trung
√ điểm DE.
3 √
Ta có: DT = cos ∠ADT · AD = a ⇒ DE = 2DT = a 3
2 √ √
Vậy chu vi tứ giác BCED là: a(1 + 2 2 + 3). 0,5 điểm
(b) ˆ Ta có ∠CED = ∠BDE ⇒ ∠CBD + ∠BDE = 180◦ ⇒ BC//DE
⇒ BCED là hình thang cân suy ra ID = IE và IB = IC
Ta có ID = IE, AD = AE ⇒ AI là đường trung trực của DE
suy ra I, A, T thẳng hàng.
1
Ta có: ∠DBE = ∠DAE = ∠DAT ⇒ BIAD nội tiếp. 0,5 điểm
2 √
3
ˆ Tứ giác BIAD nội tiếp suy ra ∠DIE = 90 ⇒ IT = DT =

a
2
a
Lại có AT = sin ∠T DA · AD =
2

Tập san Toán học STAR EDUCATION


52 NGUYỄN TẤN PHÁT


3−1
Nên AI = IT − AT = a. 0,5 điểm
2

(c) ˆ Tứ giác F BAD nội tiếp suy ra ∠DF B = 180◦ − ∠DAB = 90◦
⇒ ∠F DE = 90◦ = ∠F DB + ∠BDE ⇒ ∠F DB = 90◦ − 75◦ = 15◦
0,5 điểm
ˆ Ta có ∠IF A = ∠IDA = 45◦ − 30◦ = 15◦
Lại có ∠EAF = ∠EAC + ∠CAB + ∠BAF = 90◦ + 60◦ + 15◦ = 165◦
Suy ra ∠IF A + ∠EAF = 180◦ ⇒ AE//IF (1)
Tứ giác F IAD nội tiếp có AI//DF
⇒ F IAD là hình thang cân nên F I = AD = AE (2)
Từ (1) và (2) suy ra AEIF là hình bình hành
KA
Suy ra K là trung điểm AI nên =1 0,5 điểm
KI

Tập san Toán học STAR EDUCATION


Đề thi thử Toán Chuyên
tuyển sinh 10 THPT lần 1

Tập thể GV chuyên Star Education THCS

STAR-EDUCATION đề thi thử vào lớp 10


www.star-education.net năm học 2020-2021
môn thi: TOÁN CHUYÊN
——————

Thời gian làm bài: 150 phút


——————

Bài 1. (1,5 điểm)

(a) Cho f (x) = x2 − ax + a2 − 4, trong đó a là tham số. Tìm giá trị của
a, sao cho phương trình f (x) = 0 có hai nghiệm thực x1 và x2 sao cho
|x31 − x32 | ≤ 4.

1+3 x
(b) Giải phương trình: √ − 1 = 0.
4x + 2 + x
√ √  √
Bài 2. (1,5 điểm) Cho x, y > 0 thỏa mãn 2y > x và 11 x + y + 4 xy = 26
1 1
(a) Tìm giá trị nhỏ nhất của biểu thức: T = 11 (x + y) + + + 2021
x y
1
(b) Chứng minh rằng: + x2 + y 2 ≥ 3
x3 (2y − x)

Bài 3. (1,0 điểm) Cho hàm số bậc hai f (x) = ax2 + bx + c, (a 6= 0). Biết rằng phương
trình f (x) = x vô nghiệm. Chứng minh rằng phương trình f (f (x)) = x cũng
vô nghiệm.

Bài 4. (1,5 điểm) Cho x, y ∈ N thỏa mãn: 3x + 171 = y 2 .


.
(a) Chứng minh rằng: x .. 2.
(b) Tìm các cặp số x, y thỏa mãn phương trình.

Bài 5. (3,0 điểm) Cho đường tròn (O) và điểm P nằm ngoài đường tròn. Vẽ các tiếp
tuyến P A, P B đến (O) với A, B là các tiếp điểm. C là điểm trên cung nhỏ
AB, tiếp tuyến tại C cắt P A, P B và P O lần lượt tại D, E, F .

53
54 LƯƠNG XUÂN VINH

(a) Gọi H là giao điểm của đường tròn ngoại tiếp tam giác P DE và P O,
kéo dài HC cắt đường tròn P DE tại điểm G. Chứng minh rằng tứ giác
P F CG nội tiếp.
(b) Gọi I là tâm đường tròn nội tiếp tam giác 4P DE. Chứng minh rằng tứ
giác DOEI nội tiếp.
(c) Chứng minh rằng H là tâm đường tròn ngoại tiếp tam giác 4DOE.
(d) Chứng minh rằng đường tròn ngoại tiếp các tam giác P AB, P DE và
P CF cùng đi qua một điểm khác P .

Bài 6. (1,5 điểm) Trên mặt phẳng cho 17 điểm, trong đó không có ba điểm nào thẳng
hàng. Qua hai điểm bất kì ta vẽ được một đoạn thẳng và trên đoạn thẳng đó
ghi một số nguyên dương (các số ghi trên các đoạn thẳng khác nhau là các
số nguyên dương khác nhau). Ta tô màu mỗi đoạn thẳng bằng một trong ba
màu: đỏ, xanh và vàng.

(a) Chứng minh rằng tồn tại một tam giác có ba cạnh cùng màu.
(b) Chứng minh rằng tồn tại một tam giác có các cạnh là các đoạn thẳng đã
vẽ và tổng các số ghi trên các cạnh của tam giác đó là hợp số.

– HẾT –

Chú ý: Cán bộ coi thi không giải thích gì thêm.

Tập san Toán học STAR EDUCATION


LƯƠNG XUÂN VINH

STAR-EDUCATION đề thi thử vào lớp 10


www.star-education.net năm học 2020-2021
môn thi: TOÁN CHUYÊN
——————

Thời gian làm bài: 150 phút


——————

Bài 1. (a) Để phương trình có hai nghiệm thực x1  và x2 thì ∆ = a2 − 4 (a2 − 4) =


x1 + x2 = a
16 − 3a2 ≥ 0. Theo định lý Vietè ta có: , do đó:
x 1 x 2 = a2 − 4

x1 − x32 = |x1 − x2 | (x1 + x2 )2 − x1 x2 = |x1 − x2 | a2 − a2 + 4 = 4 |x1 − x2 | ≤ 4


3    

Lại có:
q √
0 ≤ |x1 − x2 | = (x1 + x2 )2 − 4x1 x2 = a2 − 4 (a2 − 4) = 16 − 3a2 ≤ 1
p

" √ # " √ #
4 3 √ √ 4 3
Vì vậy, ta có: a ∈ − ,− 5 ∪ 5; .
3 3

(b) ĐK: x ≥ 0. Phương trình đã cho tương đương:


√ √
1 + 3 x −√4x − 2 + x = 0

⇔ 3 x − 2 + x = 4x − 1 √
√ 
⇔ (8x − 2) = (4x − 1)
√ 3 x + 2 + x
√  
⇔ (4x
 − 1) 3 x + 2 + x − 2 = 0
4x − 1 =
√0
⇔ √
3 x+ 2+x=2
( √ )
1 7−3 5
Từ đó ta tính được hai nghiệm của phương trình là: S = ; .
4 8

Bài 2. Áp dụng bất đẳng thức Cauchy ta có:



 
x+y+2 p √  √
11 +2 (x + y) ≥ 11 2 (x + y)+2 (x + y) ≥ 11 x + y +4 xy = 26
2
15
Do đó: (x + y) ≥ 15 ⇔ x + y ≥ 2
2

(a) Áp dụng bất đẳng thức Cauchy ta có:


1 1 4
T = 11 (x + y) + + + 2021 ≥ 11 (x + y) + + 2021
x y x+y
4
= (x + y) + + 10 (x + y) + 2021
r x+y
4
≥ 2 (x + y) . + 10.2 + 2021 = 2045
(x + y)

Tập san Toán học STAR EDUCATION


56 LƯƠNG XUÂN VINH

(b) Áp dụng bất đẳng thức Cauchy ta có:


1 1 1
+ x2 + y 2 = 2 + x2 + y 2 ≥ 2 + 2xy
x3
(2y − x) 2
x (2xy − x ) r x (2xy − x2 )
1 1
= 2 2
+ x2 + (2xy − x2 ) ≥ 3 3 2 .x2 . (2xy − x2 ) = 3
x (2xy − x ) x (2xy − x2 )

Bài 3. Do phương trình f (x) = x ⇔ ax2 +bx+c = x ⇔ ax2 +(b − 1) x+c = 0, (a 6= 0)


vô nghiệm nên ta có:

∆ = (b − 1)2 − 4ac < 0 ⇔ (b − 1)2 < 4ac

Giả sử phương trình: f (f (x)) = x có nghiệm, gọi nghiệm đó là x0 , ta có:


f (f (x0 )) = x0 ⇔ f (f (x0 )) − f (x0 ) + [f (x0 ) − x0 ] = 0
⇔ a[f (x0 )]2 + bf (x0 ) − ax20 − bx0 + [f (x0 ) − x0 ] = 0
⇔ a [f (x0 ) − x0 ] [f (x0 ) + x0 ] + b [f (x0 ) − x0 ] + [f (x0 ) − x0 ] = 0
⇔ [f (x0 ) − x0 ] [a (f (x0 ) + x0 ) + b + 1] = 0
⇔ a (f (x0 ) + x0 ) + b + 1 = 0
⇔ a2 x20 + a (b + 1) x0 + ac + b + 1 = 0

Do đó phương trình: a2 x2 + a (b + 1) x + ac + b + 1 = 0 có nghiệm nên ta có:

∆ = a2 (b + 1)2 − 4a2 (ac + b + 1) ≥ 0

Từ đó dẫn đến

(b + 1)2 − 4 (ac + b + 1) ≥ 0 ⇔ 4ac ≤ b2 − 2b − 3

Suy ra: b2 − 2b − 3 > (b − 1)2 ⇔ b2 − 2b − 3 > b2 − 2b + 1 ⇔ −4 > 0 (vô lí).


Do đó ta có điều phải chứng minh.

Bài 4. (a) Lần lượt xét x = 0, 1, 2, 3 đều không nhận được x = 1, 2, 3 là nghiệm. Do
đó ta xét x ≥ 4 và x, y là hai số nguyên dương.
Vế trái chia hết cho 9 nên vế phải chia hết cho 9, đặt: y = 3z, (z ∈ N ∗ ),
ta có phương trình: 3x−2 + 19 = z 2 .
Nhận xét: 3 ≡ −1 (mod4) nên 3n ≡ 1 (mod4), nếu n chẵn và 3n ≡
−1 (mod4), nếu n lẻ.
Giả sử: Nếu x là số lẻ thì 3x−2 + 19 ≡ 18 ≡ 2 (mod4). Do một số chính
phương chia 4 chỉ dư 0 hoặc 1 (vô lí).

(b) Do đó khi x là số chẵn thì 3x−2 + 19 ≡ 20 ≡ 0 (mod4), suy ra z là số


chẳn. Đặt: x − 2 = 2k, (k ∈ N ∗ ). Ta có phương trình:
 
32k 
+ 19 = z 2 ⇔ z 2 − 32k = 19 ⇔ z 
− 3k z + 3k = 19
z + 3k = 19 z = 10 z = 10 x=6
⇔ k ⇔ k ⇔ ⇔
z−3 =1 3 =9 k=2 y = 30

Thử lại với x = 6, y = 30 (nhận). Do đó nghiệm duy nhất của phương


trình là (x; y) = (6; 30).

Tập san Toán học STAR EDUCATION


LƯƠNG XUÂN VINH

Bài 5. (a) Ta có: ∠DP H = ∠EP H (tính chất hai tiếp tuyến cắt nhau) nên ∠DGH =
∠EGH, do đó hai cung HD và cung HE bằng nhau. Từ đó:

∠HCF = ∠HGE + ∠DEG = ∠HP D + ∠DP G = ∠HP G

Dẫn đến, tứ giác CF P G nội tiếp.

(b) Ta có: ∠ODI + ∠OEI = 90o + 90o = 180o nên tứ giác DOEI nội tiếp.

(c) Xét đường tròn (P DE), với H là điểm chính giữa cung DE và I là tâm
đường tròn nội tiếp tam giác 4P DE, tính chất quen thuộc HD = HI =
HE, do đó ta có H là tâm đường tròn ngoại tiếp tứ giác DOEI.
Từ đó, H là tâm đường tròn ngoại tiếp tam giác 4DOE.

(d) Từ câu c) ta có HO = HD = HI − HE, lại có 4HDC v 4HGD (g - g)


nên HD2 = HC.HG, do đó HO2 = HC.HG. Suy ra 4HOC v 4HGO
(c - g - c) nên ∠HGO = ∠HOC.
Lại có, ∠HGP = ∠HF C nên ∠OGP = ∠HGO + ∠HGP = ∠HOC +
∠HF C = 90o , suy ra A, G, P, B, O cùng thuộc một đường tròn.

Bài 6. (a) Gọi A là một điểm đã cho, nối A với 16 điểm còn lại được 16 đoạn
thẳng và chúng được tô bởi ba màu, Theo nguyên lý Dirichlet tồn tại
ít nhất 6 đoạn thẳng có cùng một màu. Giả sử đó là các đoạn thẳng
AB, AC, AD, AE, AF, AG có cùng màu đỏ. Xét các đoạn thẳng nối từng
cặp điểm trong 6 điểm B, C, D, E, F, G. Xảy ra các trường hợp sau:
ˆ Trường hợp 1. Tồn tại một đoạn thẳng có màu đỏ, chẳng hạn BC,
thì tam giác 4ABC có ba cạnh cùng là màu đỏ, khẳng định đúng.

Tập san Toán học STAR EDUCATION


58 LƯƠNG XUÂN VINH

ˆ Trường hợp 2. Tất cả các đoạn thẳng nối B, C, D, E, F, G chỉ có


màu xanh hoặc vàng. Ta xét 5 đoạn BC, BD, BE, BF, BG được tô
bởi hai màu thì theo nguyên lý Dirichlet tồn tại ít nhất 3 đoạn thẳng
có cùng một màu. Giả sử là BC, BD, BE cùng có màu xanh.
– Nếu trong ba đoạn thẳng CD, CE, DE có một đoạn tô màu
xanh, chẳng hạn là CD thì tam giác 4BCD có ba cạnh cùng
màu xanh, khẳng định đúng.
– Nếu trong ba đoạn thẳng CD, CE, DE không có một đoạn nào
màu xanh, thì tam giác 4CDE có ba cạnh cùng màu vàng, khẳng
định đúng.
Vậy tồn tại tam giác có ba cạnh cùng một màu.

(b) Chia mỗi số nguyên dương ghi trên các đoạn thẳng cho 3 ta được các
số dư là 0, 1, 2. Ta tô màu đoạn thẳng ghi số dư 0, 1, 2 theo thứ tự ứng
với màu đỏ, xanh, vàng. Theo kết quả trên tồn tại một tam giác có
ba cạnh cùng một màu, tức là ba số đó có cùng số dư r, chẳng hạn là
3k + r, 3h + r, 3m + r. Lúc đó tổng ba số trên ba cạnh của tam giác đó
bằng:
.
3k + r + 3h + r + 3m + r = 3 (k + h + m + r) ..3
mà 3k + r + 3h + r + 3m + r > 3 do đó 3k + r + 3h + r + 3m + r là hợp số.

Tập san Toán học STAR EDUCATION


Đề thi thử Toán Chuyên
tuyển sinh 10 THPT lần 2

Tập thể GV chuyên Star Education THCS

STAR-EDUCATION đề thi thử vào lớp 10


www.star-education.net năm học 2021-2022
Lần thứ 2 môn thi: TOÁN CHUYÊN
——————
Thời gian làm bài: 150 phút
——————
Bài 1. (2 điểm)
x2 − (3m + 1)x + 2m2 + 2m
(a) Tìm m để phương trình = 0 có hai nghiệm
√ √ x
x1 , x2 phân biệt thỏa ( x1 − m + x2 − m)4 = (2m − 1)2
 p
 px2 − y = z − 1
(b) Giải hệ phương trình y2 − z = x − 1
 √ 2
z −x=y−1

Bài 2. (1.5 điểm) Cho các số x, y, z nguyên dương thỏa x > y > z.
a) Cho (x; y; z) thỏa yz + x(x + y + z) = 2021.
Tìm giá trị nhỏ nhất của biểu thức A = (x − y)2 + (x − z)2 + (y − z)2
b) Chứng minh rằng nếu y không nhỏ hơn trung bình cộng của x và z thì
(x + y + z)(xy + yz + xz − 2) ≥ 9xyz

Bài 3. (2 điểm) Cho x, y là các số nguyên không đồng thời bằng 0 sao cho x3 + y và
x + y 3 chia hết cho x2 + y 2 .
(a) Tìm x, y nếu xy = 0.
(b) Chứng minh rằng xy 6= 0 thì x, y là nguyên tố cùng nhau.
(c) Tìm tất cả cặp số nguyên (x, y) thỏa đề bài.
Bài 4. (3 điểm) Cho tam giác ABC nhọn, có trực tâm H; AH cắt BC tại D. Trên
tia đối tia DH lấy điểm M . Đường tròn ngoại tiếp tam giác M BH cắt AB
tại E cắt BC tại K; đường tròn ngoại tiếp tam giác M CH cắt AC tại F và
BC tại L.

59
60 NGUYỄN TĂNG VŨ

(a) Chứng minh BEF C nội tiếp và ∠EM A = ∠F M A.


(b) M E cắt CH tại P , M F cắt BH tại Q. Chứng minh P Q vuông góc OA
với O là tâm đường tròn ngoại tiếp tam giác ABC.
(c) HK cắt AC tại U , HL cắt AB tại V . Chứng minh U V luôn song song
với một đường thẳng cố định khi M thay đổi.

Bài 5. (1.5 điểm) Trong một hội nghị Toán quốc tế có n người, mỗi người trong họ
có thể nói được nhiều nhất 3 ngôn ngữ. Trong 3 người bất kì thì luôn có 2
người có thể nói chung một ngôn ngữ.

(a) Chứng minh rằng nếu n ≥ 9 thì có một ngôn ngữ mà có thể nói bởi ít
nhất 3 người.
(b) Nếu n = 8, điều kết luận của câu a) còn đúng không? Tại sao?

– HẾT –

Tập san Toán học STAR EDUCATION


NGUYỄN TĂNG VŨ

STAR-EDUCATION ĐỀ THI THỬ VÀO LỚP 10


www.star-education.net hướng dẫn: TOÁN CHUYÊN

ĐÁP ÁN - THANG ĐIỂM


Bài 1. (2 điểm)
x2 − (3m + 1)x + 2m2 + 2m
(a) Tìm m để phương trình = 0 có hai nghiệm
x
x1 , x2 phân biệt thỏa
√ √
( x1 − m + x2 − m)4 = (2m − 1)2
 p
 px2 − y = z − 1
(b) Giải hệ phương trình y2 − z = x − 1
 √ 2
z −x=y−1

Lời giải.

a) (1 điểm) Phương trình đã cho có hai nghiệm phân biệt khi và chỉ khi
phương trình
x2 − (3m + 1)x + 2m2 + 2m = 0
có hai nghiệm phân biệt khác 0. Điều này tương đương với
( (
∆ = (3m + 1)2 − 4(2m2 + 2m) > 0 (m − 1)2 > 0
⇐⇒ ⇐⇒ m ∈
/ {0, ±1}.
2m2 + 2m 6= 0 m 6= 0 và m 6= −1

Ta có
√ √ 2 p
x1 − m + x2 − m = (x1 + x2 ) − 2m + 2 x1 x2 − m (x1 + x2 ) + m2

= m + 1 + 2 m.

nên điều kiện trong đề bài tương đương với


 √  √ √
m + 2√m + 1 = 2m − 1 m = 1 + 3
⇔ √ .
m + 2 m + 1 = −2m + 1 m=0
√ 2
Đối chiếu lại điều kiện của m, ta suy ra m = 1 + 3 .
b) (1 điểm) Điều kiện xác định: x, y, z ≥ 1.
Bình phương hai vế 3 phương trình của hệ rồi cộng lại theo vế, ta suy ra
x + y + z = 3, từ kết quả này, cộng vế 3 phương trình của hệ, ta có
p p √
x2 − y + y 2 − z + z 2 − x = 0,

suy ra x2 − y = y 2 − z = z 2 − x = 0. Do đó x2 + y 2 + z 2 = 3. Tuy nhiên,


chú ý rằng
3(x2 + y 2 + z 2 ) ≥ (x + y + z)2 = 9,
đẳng thức xảy ra khi và chỉ khi x = y = z nên ta suy ra x = y = z = 1.
Thử lại, ta kết luận hệ đã cho có nghiệm duy nhất x = y = z = 1

Tập san Toán học STAR EDUCATION


62 NGUYỄN TĂNG VŨ

Bài 2. (1.5 điểm) Cho các số x, y, z nguyên dương thỏa x > y > z.

a) Cho (x; y; z) thỏa yz + x(x + y + z) = 2021.


Tìm giá trị nhỏ nhất của biểu thức

A = (x − y)2 + (x − z)2 + (y − z)2

b) Chứng minh rằng nếu y không nhỏ hơn trung bình cộng của x và z thì

(x + y + z)(xy + yz + xz − 2) ≥ 9xyz

Lời giải.

a) (0.75 điểm) Ta có (x + y)(x + z) = 2021 = 43 · 47 = 1 · 2021.


Mà x + y > x + z > 2. Suy ra x + y = 47, x + z = 43.
Khi đó (x − y)2 + (x − z)2 + (y − z)2 = (2x − 47)2 + (2x − 43)2 + 16
= 8x2 − 360x + 432 + 472 + 16 = 8(x2 − 45x) + 4074.
Do x + y = 47, x > y suy ra x ≥ 24.
Suy ra x2 − 45x = (x − 24)(x − 21) − 504 ≥ −504.
Do đó A ≥ 4074 − 8 · 504 = 42.
Đẳng thức xảy ra khi x = 24, y = 23, z = 19.
Vậy giá trị nhỏ nhất của A là 42 khi x = 24, y = 23, z = 19.
b) (0,75) Ta có x ≥ y + 1 ≥ z + 2. Suy ra

(x − y)2 ≥ 1, (y − z)2 ≥ 1, (x − z)2 ≥ 4

Suy ra

x2 + y 2 ≥ 2xy + 1, y 2 + z 2 ≥ 2yz + 1, x2 + z 2 ≥ 2xz + 4

Từ đó

zx2 + zy 2 ≥ 2xyz + z, xy 2 + xz 2 ≥ 2xyz + x, yx2 + yz 2 ≥ 2xyz + 4y

Cộng lại ta có

xy(x + y) + yz(y + z) + zx(z + x) ≥ 6xyz + x + 4y + z

Suy ra (x + y + z)(xy + yz + zx − 2) ≥ 9xyz − x + 2y − z.

1
Vì y ≥ (x + z) nên ta có điều cần chứng minh.
2
Bài 3. (2 điểm) Cho x, y là các số nguyên không đồng thời bằng 0 sao cho x3 + y và
x + y 3 chia hết cho x2 + y 2 .

(a) (0.5 điểm) Tìm x, y nếu xy = 0.


(b) Chứng minh rằng xy 6= 0 thì x, y là nguyên tố cùng nhau.
(c) Tìm tất cả cặp số nguyên (x, y) thỏa đề bài.

Lời giải.

Tập san Toán học STAR EDUCATION


NGUYỄN TĂNG VŨ

a) (0.5 điểm) Nếu xy = 0, giả sử y = 0 thì x3 chia hết cho x2 (hiển nhiên)
và x chia hết cho x2 nên nếu x 6= 0 thì |x| ≥ x2 , do đó x = ±1. Từ đó suy
ra tất cả các số nguyên x, y thỏa mãn đề bài là (x, y) = (0, ±1), (±1, 0).
b) (0.75 điểm) Đặt gcd (x, y) = d thì x = da, y = db, gcd(a, b) = 1, a, b ∈
Z \ {0}. Ta có d2 | x2 + y 2 nên d2 | x + y 3 = da + d3 b3 nên d | a. Tương
tự d | b nên d = 1.
c) (0.75 điểm) Từ ý a), ta chỉ cần xét trường hợp xy 6= 0. Lúc này, ta có
gcd(x, y) = 1 và x2 + y 2 | x3 + y, x2 + y 2 | x(x2 + y 2 ) nên
x2 + y 2 | y − xy 2 = y (1 − xy) .
Mà gcd(x, y) = 1 nên ta suy ra x2 + y 2 | 1 − xy. Từ đó ta có
|xy| + 1 ≥ |xy − 1| ≥ x2 + y 2 ≥ 2xy
nên |xy| ≤ 1. Mà xy 6= 0 nên |xy| = 1. Từ đó ta tìm được (x, y) = (1, 1),
(1, −1), (−1, 1), (−1, −1).
Bài 4. (3 điểm) Cho tam giác ABC nhọn, có trực tâm H; AH cắt BC tại D. Trên
tia đối tia DH lấy điểm M . Đường tròn ngoại tiếp tam giác M BH cắt AB
tại E cắt BC tại K; đường tròn ngoại tiếp tam giác M CH cắt AC tại F và
BC tại L.
(a) Chứng minh BEF C nội tiếp và ∠EM A = ∠F M A.
(b) M E cắt CH tại P , M F cắt BH tại Q. Chứng minh P Q vuông góc OA
với O là tâm đường tròn ngoại tiếp tam giác ABC.
(c) HK cắt AC tại U , HL cắt AB tại V . Chứng minh U V luôn song song
với một đường thẳng cố định khi M thay đổi.
Lời giải.
a) (1 điểm) Ta có tứ giác EBM H nội tiếp nên ∠AEH = ∠AM B, suy ra
hai tam giác AEH và AM B đồng dạng, kéo theo AE · AB = AH · AM .
Chứng minh tương tự ta thu được AF · AC = AH · AM = AE · AB, từ
đó dễ dàng có được hai tam giác AEF và ABC đồng dạng (cạnh - góc
- cạnh) nên ∠AEF = ∠ACB, suy ra BEF C nội tiếp. Ta có HM BE và
HM CF là các tứ giác nội tiếp nên
∠HM E = ∠HBE = 90◦ − ∠A = ∠HCF = ∠HM F .
b) (1 điểm) Ta có ∠AEF = ∠ACB = 90◦ − ∠BAO nên AO ⊥ EF . Vì vậy
ta chỉ cần chứng minh P Q k EF .
Thật vậy, ta có M H là phân giác góc P M Q và
1
∠P HQ = 90◦ + ∠ACH = 90◦ + ∠HM Q = 90◦ + ∠P M Q
2
nên H là tâm đường tròn nội tiếp tam giác P M Q. Do đó
∠P QM = 2∠P HM − 180◦ = 180◦ − 2∠M HC = 180◦ − ∠ABC.
Lại có
∠EF M = ∠EF C −∠M F C = 180◦ −∠ABC −∠M HC = 180◦ −2∠ABC.
Suy ra ∠P QM = ∠EF M nên P Q k EF , kéo theo P Q ⊥ AO.

Tập san Toán học STAR EDUCATION


64 NGUYỄN TĂNG VŨ

c) (1 điểm) Ta có

∠HU F = ∠U KB − ∠ACB = ∠AEH − ∠AEF = ∠F EH

nên F HEU nội tiếp. Chứng minh tương tự ta cũng có F HEV nội tiếp
nên U V F E nội tiếp. Suy ra ∠F U V = ∠V EF = ∠F CB nên U V k BC.
Vậy U V luôn luôn song song với BC cố định.

Bài 5. (1.5 điểm) Trong một hội nghị Toán quốc tế có n người, mỗi người trong họ
có thể nói được nhiều nhất 3 ngôn ngữ. Trong 3 người bất kì thì luôn có 2
người có thể nói chung một ngôn ngữ.

(a) Chứng minh rằng nếu n ≥ 9 thì có một ngôn ngữ mà có thể nói bởi không
ít hơn 3 người.
(b) Nếu n = 8, điều kết luận của câu a) còn đúng không? Tại sao?

Lời giải.

a) Giả sử mọi ngôn ngữ đều được bởi nhiều nhất là 2 người.
ˆ Khi đó mỗi người có thể giao tiếp được với nhiều nhất 3 người khác
với 3 ngôn ngữ khác nhau. Tức là A có thể nói với nhiều nhất B, C, D
với ngôn ngữ khác nhau. Nếu ngược lại A nói thêm với E thì theo
nguyên lí Dirichlet sẽ nói cùng ngôn ngữ với A hoặc B hoặc C. Khi
đó có một ngôn ngữ có 3 người nói, mâu thuẫn.
ˆ Ta có A nói với B, C, D với 3 ngôn ngữ khác nhau. Xét người E thì E
cùng giao tiếp với 3 người là F, G, H. Do đó còn một người X ngoài
nhóm này. Khi đó A, E, X không nói chung ngôn ngữ nào. Vô lý.
b) Kết luận không còn đúng, ví dụ: chia 8 người thành 2 nhóm, A, B, C, D
và E, F, G, H. Hai người trong cùng nhóm nói với nhau, mỗi cặp nói ngôn
ngữ khác nhau. Khi đó sẽ thỏa đề bài. Xem hình vẽ:

F A
7
E 1
8 9
D 2 3
12
G 5
11 6
10
B 4 C
H

Tập san Toán học STAR EDUCATION


Đề kiểm tra cuối khóa lớp 10CĐ

Tập thể GV chuyên Star Education THPT

STAR-EDUCATION MÔN: TOÁN - LỚP 10 CĐ


KIẾM TRA CUỐI KHÓA năm học 2020-2021
(Đề chính thức) Thời gian: 180 phút (cho mỗi phần bên dưới)

Bài 1. (4 điểm) Cho dãy số (xn ) xác định bởi x1 = 1 và

xn (6 + x2n )
xn+1 = ,
3x2n + 2

với mọi n > 1. Hãy xét tính đơn điệu và bị chặn của (xn ).
Bài 2. (4 điểm) Cho a, b, c là các số thực không âm thỏa mãn ab + bc + ca = 1.
Chứng minh rằng
1 1 1
+ 2 + 2 > 1.
4a2 − bc + 2 4b − ca + 2 4c − ab + 2

Bài 3. (4 điểm)
a) Tìm tất cả các cặp số nguyên (x; y) thỏa mãn

x2 (y − 1) + y 2 (x − 1) = 1.

b) Tìm tất cả các cặp số nguyên dương (x; y) thỏa mãn

x19 − 1 = (x − 1) y 12 − 1 .


Bài 4. (5 điểm) Cho tứ giác ABCD nội tiếp có E là giao điểm của AB và CD, F
là giao điểm của AD và BC.
a) Chứng minh rằng đường tròn ngoại tiếp các tam giác EAD, EBC, F AB,
F CD cùng đi qua một điểm P .

b) Tiếp tuyến tại P của (F AB) cắt AB tại X. Tiếp tuyến tại P của (EBC) cắt
BC tại Y . Tiếp tuyến tại P của (F CD) cắt CD tại Z. Tiếp tuyến tại P của
(EDA) cắt DA tại T . Chứng minh rằng X, Y , Z, T thẳng hàng.

65
66 NGUYỄN TIẾN HOÀNG - NGUYỄN CÔNG THÀNH

c) Chứng minh rằng đường tròn ngoại tiếp các tam giác P XZ và P Y T tiếp xúc
nhau.

Bài 5. (3 điểm) Cho tập hợp X có n phần tử và m tập con khác rỗng T1 , T2 , . . . , Tm
thỏa mãn

i) |Ti | = 4, i = 1, m.

ii) Với mọi i, j ∈ X thì tồn tại duy nhất k sao cho (i, j) ⊂ Tk .

iii) Với mọi 1 6 i < j 6 m thì |Ti ∩ Tj | = 1.

Tìm tất cả các giá trị có thể có của m, n.

Tập san Toán học STAR EDUCATION


NGUYỄN TIẾN HOÀNG - NGUYỄN CÔNG THÀNH

LỜI GIẢI CHI TIẾT

Ví dụ 1 (4 điểm). Cho dãy số (xn ) xác định bởi x1 = 1 và

xn (6 + x2n )
xn+1 = ,
3x2n + 2

với mọi n > 1. Hãy xét tính đơn điệu và bị chặn của (xn ). 

Lời giải. Ta thấy (xn ) là dãy dương và với mỗi n nguyên dương, ta có
xn (6 + x2n ) 2 − x2n
xn+1 − xn = − x n = 2x n · . (1)
3x2n + 2 3x2n + 2

Tuy nhiên 1 = x1 < 2 và
√ 3
√ xn (6 + x2n ) √ xn − 2
xn+1 − 2= − 2=
3x2n + 2 3x2n + 2

nên bằng phương pháp quy nạp, ta dễ dàng chứng minh được xn < 2 với mọi n
nguyên dương. Vì vậy từ (1) ta suy ra xn+1 > xn hay (xn ) tăng.
Vậy (xn ) tăng và bị chặn.

Ví dụ 2 (4 điểm). Cho a, b, c là các số thực không âm thỏa mãn ab + bc + ca = 1.


Chứng minh rằng
1 1 1
+ 2 + 2 > 1.
4a2 − bc + 2 4b − ca + 2 4c − ab + 2


Lời giải. Bất đẳng thức cần chứng minh là hiển nhiên khi abc = 0 nên ta chỉ cần
xét trường hợp abc > 0, lúc này áp dụng bất đẳng thức AM - GM, ta có

2 (c(2a + b) + b(2a + c))2


4a − bc + 2(ab + bc + ca) = (2a + b)(2a + c) 6
4bc
(ab + bc + ca)2 1
= = ,
bc bc
suy ra
1
2
> bc.
4a − bc + 2
Thực hiện các bất đẳng thức tương tự rồi cộng theo vế ta thu được điều cần chứng
minh.

Ví dụ 3 (4 điểm).

a) Tìm tất cả các cặp số nguyên (x; y) thỏa mãn

x2 (y − 1) + y 2 (x − 1) = 1.

Tập san Toán học STAR EDUCATION


68 NGUYỄN TIẾN HOÀNG - NGUYỄN CÔNG THÀNH

b) Tìm tất cả các cặp số nguyên dương (x; y) thỏa mãn

x19 − 1 = (x − 1) y 12 − 1 .


Lời giải. a) Đặt s = x + y và p = xy. Phương trình trở thành

sp − (s2 − 2p) = 1

hay có thể viết lại thành:

(s + 2)(s − 2 − p) = −5

Lập bảng giá trị của s và p, ta có:

s+2 −5 −1 1 5
s−2−p 1 5 −5 −1
s −7 −3 −1 3
p −10 −10 2 2

Do s2 −4p ≥ 0 nên nhận bộ (s, p) ∈ {(−7, −10); (−3, −10); (3, 2)}. Từ đây cùng
điều kiện x, y là số nguyên, giải được (x, y) ∈ {(−5, 2); (2, −5); (1, 2); (2, 1)}.
Thử lại, đây là tất cả các nghiệm của bài toán.

b) Nếu x = 1 thì y là số nguyên dương bất kỳ. Xét x ≥ 2.


x19 − 1
Viết lại phương trình đã cho thành = y 12 − 1. Trước hết ta chứng
x−1
x19 − 1
minh rằng mọi ước nguyên tố của hoặc là 19, hoặc là số nguyên tố p
x−1
thoả mãn p ≡ 1 (mod 19).
x19 − 1
Thật vậy, gọi p 6= 19 là một ước nguyên tố của , khi đó x19 ≡ 1 (mod
x−1
p). Theo định lý Fermat nhỏ thì xp−1 ≡ 1 (mod p), do đó x(19,p−1) ≡ 1 (mod
p). Có hai khả năng: nếu (19, p − 1) = 19 thì p ≡ 1 (mod 19). Khả năng còn
lại, nếu (19, p − 1) = 1 thì x ≡ 1 (mod p). Tuy nhiên từ đó thì:

x19 − 1
0≡ ≡ x18 + x17 + · · · + 1 ≡ 19 (mod p)
x−1

Đây là điều mâu thuẫn. Do đó chỉ xảy ra khả năng p ≡ 1 (mod 19).
Bây giờ lại có hai khả năng cần xem xét:
x19 − 1
ˆ Nếu không là bội của 19, từ kết quả trên thì y 12 − 1 ≡ 1 (mod
x−1
19). Chuyển vế và luỹ thừa, ta được y 36 ≡ 8 (mod 19). Do 19 là số nguyên
tố và (y, 19) = 1 nên theo định lý Fermat nhỏ:

y 36 ≡ (y 18 )2 ≡ 1 (mod 19), mâu thuẫn.

Tập san Toán học STAR EDUCATION


NGUYỄN TIẾN HOÀNG - NGUYỄN CÔNG THÀNH

x19 − 1
ˆ Nếu là bội của 19, ta có y 12 ≡ 1 (mod 19). Từ đây y (12,18) ≡ 1
x−1
(mod 19) hay y 6 ≡ 1 (mod 19). Hệ quả là y 6 + 1 ≡ 2 (mod 19), nghĩa là
y 12 − 1 có ước nguyên tố không đồng dư với 1 (mod 19), mâu thuẫn.
Kết luận: tất cả các nghiệm của phương trình là (1, y) với y ∈ Z+ .

Ví dụ 4 (5 điểm). Cho tứ giác ABCD nội tiếp có E là giao điểm của AB và


CD, F là giao điểm của AD và BC.

a) Chứng minh rằng đường tròn ngoại tiếp các tam giác EAD, EBC, F AB,
F CD cùng đi qua một điểm P .

b) Tiếp tuyến tại P của (F AB) cắt AB tại X. Tiếp tuyến tại P của (EBC)
cắt BC tại Y . Tiếp tuyến tại P của (F CD) cắt CD tại Z. Tiếp tuyến tại
P của (EDA) cắt DA tại T . Chứng minh rằng X, Y , Z, T thẳng hàng.

c) Chứng minh rằng đường tròn ngoại tiếp các tam giác P XZ và P Y T tiếp
xúc nhau.

Y
P

T
B

E C
Z D

Lời giải. Để chính xác, lời giải dưới đây sử dụng góc định hướng.
a) Gọi P là giao điểm khác E của (EAB) và (ECD). Ta có:
(P A, P F ) ≡ (P A, P E) ≡ (DA, DE)
≡ (DA, DC) ≡ (BA, BC) ≡ (BA, BF ) (mod π)
Do đó P ∈ (F AD). Tương tự, ta cũng có:
(P D, P F ) ≡ (P D, P E) ≡ (AD, AE)
≡ (AD, AB) ≡ (CD, CB) ≡ (CD, CF ) (mod π)
Do đó P ∈ (F BC) nên (EAD), (EBC), (F AB) và (F CD) cùng đi qua P .

Tập san Toán học STAR EDUCATION


70 NGUYỄN TIẾN HOÀNG - NGUYỄN CÔNG THÀNH

b) Nhờ các tứ giác nội tiếp nên:

(P E, P F ) ≡ (P E, P A) + (P A, P F ) ≡ (DE, DA) + (BA, BF )


≡ (DC, DA) + (BA, BC) ≡ (DC, DA) + (DA, DC) ≡ 0 (mod π)

Do đó E, P, F thẳng hàng. Vì P X, P T lần lượt là tiếp tuyến của (F AB) và


(EAD) nên:

(P X, P T ) ≡ (P X, P A) + (P A, P T ) ≡ (F P, F A) + (EA, EP )
≡ (F E, F A) + (EA, EF ) ≡ (AE, AF ) ≡ (AX, AT ) (mod π)

Do đó A, P, T, X cùng thuộc một đường tròn nên (XP, XT ) ≡ (AP, AT ) ≡


(AP, AF ) (mod π). Đồng thời ta cũng có:

(P X, P Y ) ≡ (P X, P B) + (P B, P Y ) ≡ (F P, F B) + (EB, EP )
≡ (F E, F B) + (EB, EF ) ≡ (BE, BF ) ≡ (BX, BY ) (mod π)

Do đó B, P, X, Y cùng thuộc một đường tròn. Hệ quả là:

(XP, XY ) ≡ (BP, BY ) ≡ (BP, BF ) ≡ (AP, AF ) ≡ (XP, XT ) (mod π)

Từ đó T, X, Y thẳng hàng. Chứng minh tương tự, ta cũng có các điểm P, D, Z, T


thuộc cùng một đường tròn và:

(T P, T Z) ≡ (DP, DZ) ≡ (DP, DE) ≡ (AP, AE) ≡ (T P, T X) (mod π)

Hệ quả là T, X, Z thẳng hàng. Vậy X, Y, Z, T thẳng hàng.

c) Gọi d là tiếp tuyến của (P XZ) tại P . Từ các tứ giác nội tiếp thì:

(d, P T ) ≡ (P X, P T ) + (d, P X) ≡ (AX, AT ) + (ZP, ZX)


≡ (AE, AF ) + (ZP, ZT ) ≡ (EA, EF ) + (EF, AF ) + (DT, DP )
≡ (EA, EP ) + (F P, F A) + (DA, DP ) ≡ (BP, BA)
≡ (BP, BX) ≡ (Y P, Y X) ≡ (Y P, Y T ) (mod π)

Từ đó d tiếp xúc với (P Y T ). Vậy (P XZ) và (P Y T ) tiếp xúc nhau.

Ví dụ 5 (3 điểm). Cho tập hợp X có n phần tử và m tập con khác rỗng


T1 , T2 , . . . , Tm thỏa mãn

i) |Ti | = 4, i = 1, m.

ii) Với mọi i, j ∈ X thì tồn tại duy nhất k sao cho (i, j) ⊂ Tk .

iii) Với mọi 1 6 i < j 6 m thì |Ti ∩ Tj | = 1.

Tìm tất cả các giá trị có thể có của m, n. 

Tập san Toán học STAR EDUCATION


NGUYỄN TIẾN HOÀNG - NGUYỄN CÔNG THÀNH

Lời giải. Ta sẽ tính số bộ (i, j, T ) mà (i, j) ⊂ T .


Xét các cặp (i, j) ⊂ X mà i 6= j thì có tất cả Cn2 cặp. Mà |Ti | = 4 nên với mỗi tập
n (n − 1)
Ti thì có sự lặp lại C42 = 6 lần, suy ra m = .
2
Đặt T1 = {a1 , a2 , a3 , a4 } là một tập hợp nào đó trong các tập hợp đã cho. Xét ak ∈ X
mà ak ∈ / T1 , với mỗi bộ (ak , ai ) thì ta có 1 tập Tj mà mỗi tập tính 3 lần, suy ra số
n−4 4 (n − 4)
tập là , do đó m = 1 + . Từ đó ta có
3 3
n(n − 1) 4(n − 4)
=1+ ⇒ (n − 4)(n − 13) = 0.
12 3
Từ đó ta tìm được n = 4, m = 1 và n = m = 13.
Cuối cùng là việc xây dựng ví dụ:
ˆ Nếu (m, n) = (1, 4) thì chọn T1 = X.
ˆ Nếu (m, n) = (13, 13), đặt X = {a1 , a2 , · · · , a13 } và xét cách xây dựng sau:

Trong đó ô vuông (i, j) được tô màu nghĩa là ai ∈ Tj .


Kết luận: các cặp số (m, n) có thể có là (1, 4) và (13, 13).

Tập san Toán học STAR EDUCATION


Lời giải các câu VD-VDC
đề minh họa THPT quốc gia 2021

Lê Phúc Lữ
(GV Đại học KHTN TPHCM)

1. Phân tích
Tham khảo đề thi đầy đủ tại link bên dưới
https://moet.gov.vn/tintuc/Pages/tin-tong-hop.aspx?ItemID=7275

1.1. Cấu trúc đề


Ta xét số câu theo từng chương như sau (có tính thêm bốn mức độ: nhận biết, thông
hiểu, vận dụng và vận dụng cao).

1. Chương Ứng dụng đạo hàm: 10 = 4 + 4 + 1 + 4.

2. Chương Hàm số lũy thừa, mũ & logarit: 8 = 3 + 3 + 1 + 1.

3. Chương Nguyên hàm & tích phân: 3 + 2 + 1 + 1.

4. Chương Số phức: 6 = 2 + 2 + 1 + 1.

5. Chương Thể tích khối đa diện: 3 = 1 + 1 + 1 + 0.

6. Chương Khối tròn xoay: 3 = 1 + 1 + 1.

7. Chương Hình giải tích trong không gian: 8 = 3 + 3 + 1 + 1.

8. Phần Đại số lớp 11 (tổ hợp, xác suất, dãy số): 3 = 1 + 2 + 0 + 0.

9. Phần Hình học lớp 11 (khoảng cách, góc): 2 = 0 + 2 + 0 + 0.

1.2. Nhận xét


Ta có các nhận xét sau đây:

ˆ Các câu khó, mức độ 4 thuộc về các phần: (1), (2), (3), (4), (7).

ˆ Các câu mức độ 3 có khoảng 10 câu và có đủ ở các phần, còn lại 35 câu mức
1 − 2.

72
LÊ PHÚC LỮ

ˆ Nội dung của lớp 11 chiếm 10%, các câu mức độ 1 − 2.

ˆ Các câu ở mỗi mức độ đang được sắp xếp theo từng chương (giống năm 2017),
nhưng đề chính thức chắc không như thế.

ˆ So về mức độ thì đề này dễ hơn đề chính thức năm 2019 nhưng khó hơn đề
năm 2020.

ˆ Không có xuất hiện phần: lượng giác, bài toán vận tốc, bài toán lãi suất,
phương trình tiếp tuyến, khoảng cách đường chéo nhau.

ˆ Về 5 câu khó nhất (vận dụng cao): câu 46, biện luận số cực trị của hàm chứa
trị tuyệt đối là khó nhất đề, đòi hỏi thực hiện nhiều bước; câu 47, 48, 49 đòi
hỏi có các kinh nghiệm nhất định ở dạng này để chọn hướng tiếp cận đúng
mới xử lý nhanh gọn được; câu 50 có nét mới là kết hợp nhiều chương: khối
tròn xoay, tìm giá trị lớn nhất và hình giải tích Oxyz.

ˆ Thời gian lý tưởng để học sinh muốn được điểm 9+ ở đề này là: 38 câu đầu
làm (và kiểm tra lại) trong 25 phút; 7 câu tiếp theo làm trong 25 − 35 phút;
5 câu cuối dành 30 − 40 phút còn lại để làm được càng nhiều càng tốt.

2. Lời giải các câu VD-VDC

Lời giải. Chọn câu C.


Đặt 2x = t thì t ∈ [−3; 4] và ta đưa về xét h(t) = f (t) − 2t. Ta có h0 (t) = f 0 (t) − 2
nên dựa vào đồ thị đã cho thì h0 (t) = 0 có hai nghiệm t = 0, t = 2, trong đó f 0 (t) − 2
lại không đổi dấu khi qua t = 0, còn h0 (t) đổi dấu từ + sang − khi qua t = 2. Lập
bảng biến thiên cho h(t) trên [−3; 4], ta có max h(t) = h(2) = f (2) − 4.

Lời giải. Chọn câu A.



Đặt t = 2x > 0 thì ta có bất phương trình (2t − 2)(t − y) < 0 hay

2
(t − )(t − y) < 0 (∗).
2

Tập san Toán học STAR EDUCATION


74 LÊ PHÚC LỮ


2
Vì y ∈ Z+ nên y > 2
, do đó
√ √
2 2 1
(∗) ⇔ <t<y⇔ < 2x < y ⇔ − < x < log2 y.
2 2 2

Nếu log2 y > 10 thì x ∈ {0, 1, 2, . . . , 10} đều là nghiệm, không thỏa. Suy ra log2 y ≤ 10
hay y ≤ 210 = 1024, từ đó có y ∈ {1, 2, . . . , 1024}.

Lời giải. Chọn câu B.


Trong tích phân I đã cho, đặt t = 2 sin x + 1 thì dt = 2 cos xdx. Ta có
Z 3 Z 2 Z 3
1 1 2 1 23
I= f (t)dt = (t − 2t + 3)dt + (t2 − 1)dt = .
2 1 2 1 2 1 6

Lời giải. Chọn câu C.


Đặt z = a + bi với a, b ∈ R thì (z + 2i)(z − 2) = (a + (b + 2)i)(a − 2 − bi) =
a(a − 2) + b(b + 2). Do đó, ta có hệ
(
a2 + b 2 = 2
a(a − 2) + b(b + 2) = 0

hay (
a2 + b 2 = 2
.
a−b=1

Giải hệ này được hai nghiệm.

Tập san Toán học STAR EDUCATION


LÊ PHÚC LỮ

Lời giải. Chọn câu A.


Gọi M là trung điểm BC thì AM ⊥BC và SA⊥BC nên BC⊥(SAM ). Từ đây dễ
thấy

\ = 45◦ . Do đó, SAM vuông cân ở A và SA = AM =
góc cần tìm là α = ASM
a 3
. Suy ra
2 √ √
1 a 3 a2 3 a3
VS.ABC = · · = .
3 2 4 8

Lời giải. Chọn câu C.


Gọi r là bán kính đáy của hình trụ thì ta có
4, 45 = 2r · sin 150◦ ⇒ r = 4, 45.
Từ đó suy ra góc ở tâm ứng với cung này là 60◦ và cung này bằng 1
6
chu vi đường
tròn đáy.
Ta có diện tích xung quanh của các hình trụ là Sxd = 2πrh nên diện tích của tấm
kính chính là 16 · 2πrh = πrh
3
. Do đó, giá tiền là
π · 4, 45 · 1, 35
1.500.000 × ≈ 9.437.000(V N D).
3

Lời giải. Chọn câu A.


Gọi A(2a + 1, a, −2a − 1) và B(b + 2, 2b, −b − 1) lần lượt là giao điểm của đường
−→
thẳng d cần tìm với d1 , d2 . Ta có AB = (b − 2a + 1, 2b − a, −b + 2a) nên d⊥(P ) thì
b − 2a + 1 2b − a −b + 2a
= = .
2 2 −1
−→
Giải ra được (a; b) = (0; 1) nên AB = (2; 2; −1) và A(1; 0; −1), B(3; 2; −2).Từ đó
viết được (d) : x−3
2
= y−2
2
= z+2
−1
.

Tập san Toán học STAR EDUCATION


76 LÊ PHÚC LỮ

Lời giải. Chọn câu A.


Ta có f 0 (x) bậc ba có 2 điểm cực trị là x = −3, x = −1 nên

f 00 (x) = a(x + 3)(x + 1).


3
Suy ra f 0 (x) = a( x3 + 2x2 + 3x) + b. Từ f (−3) = −1 và f (−1) = − 613
, giải ra
29 0 29 x3 2 0
a = 2 , b = −1 hay f (x) = 2 ( 3 + 2x + 3x) − 1. Do đó f (0) = −1 < 0.
Đặt h(x) = f (x3 ) − 3x thì h0 (x) = 3x2 f 0 (x3 ) − 3 nên h0 (x) = 0 ⇔ f 0 (x3 ) = x12 . (∗)
Trên (−∞; 0) thì f 0 (x) < 0 nên f 0 (x3 ) < 0, ∀x < 0, kéo theo (∗) vô nghiệm trên
(−∞; 0].
Xét x > 0 thì f 0 (x) đồng biến còn x12 nghịch biến nên (∗) có không quá 1 nghiệm. Lại
có lim+ (f 0 (x3 ) − x12 ) = −∞ và lim (f 0 (x3 ) − x12 ) = +∞ nên (∗) có đúng nghiệm
x→0 x→+∞
x = c > 0. Từ bảng biến thiên của h(x), thì để ý h(0) = f (0) = 0 nên h(c) < 0 và
phương trình h(x) = 0 có hai nghiệm thực phân biệt, khác c.
Từ đó |h(x)| sẽ có 3 điểm cực trị.

Lời giải. Chọn câu A.


Điều kiện x > 0. Đặt y = alog x + 2 > 0 thì y log a = x − 2 ⇔ alog y + 2 = x. Từ đó ta
có hệ (
y = alog x + 2
.
x = alog y + 2
Do a ≥ 2 nên hàm số f (t) = at +2 là đồng biến trên R. Giả sử x ≥ y thì f (y) ≥ f (x)
sẽ kéo theo y ≥ x, tức là phải có x = y. Tương tự nếu x ≤ y.
Vì thế, ta đưa về xét phương trình x = alog x + 2 với x > 0 hay x − xlog a = 2. Ta
phải có x > 2 và
x > xlog a ⇔ 1 > log a ⇔ a < 10.
Ngược lại, với a < 10 thì xét hàm số liên tục g(x) = x − xlog a − 2 = xlog a (x1−log a −
1) − 2 có lim g(x) = +∞ và g(2) < 0. nên g(x) sẽ có nghiệm trên (2; +∞). Do đó,
x→+∞
mọi số a ∈ {2, 3, . . . , 9} đều thỏa mãn.

Tập san Toán học STAR EDUCATION


LÊ PHÚC LỮ

Lời giải. Chọn câu D.


Rõ ràng kết quả bài toán không đổi nếu ta tịnh tiến đồ thị sang trái cho điểm uốn
trùng gốc tọa độ O. Gọi g(x) = ax3 + bx2 + cx + d là hàm số khi đó thì dễ thấy g(x)
lẻ nên có ngay b = d = 0 và g(x) = ax3 + cx có hai điểm cực trị tương ứng là −1, 1,
cũng là nghiệm của 3ax2 + c = 0. Từ đó dễ dàng có g(x) = k(x3 − 3x) với k > 0.
Xét diện tích hình chữ nhật

S1 + S2 = |(−1) · g(−1)| = 2k.


R0 S1
Ngoài ra, S2 = k −1
|x3 − 3x| dx = 45 k. Vì thế S1 = 2k − 5k
4
= 3k
4
và S2
= 35 .

Lời giải. Chọn câu B.


Đặt z1 = a + bi, z2 = c + di với a, b, c, d ∈ R. Theo giả thiết thì

a2 + b2 = 1, c2 + d2 = 4, (a − c)2 + (b − d)2 = 3.

Do đó
a2 − 2ac + c2 + b2 − 2bd + d2 = 3 ⇒ ac + bd = 1.
Ta có 3z1 + z2 = 3(a + c) + (3b + d)i nên

|3z1 + z2 | = (3a + c)2 + (3b + d)2 = 9(a2 + b2 ) + (c2 + d2 ) + 6(ac + bd) = 19.

Áp dụng bất đẳng thức |z + z 0 | ≤ |z| + |z 0 |, ta có ngay



|3z1 + z2 − 5i| ≤ |3z1 + z2 | + |−5i| = 19 + 5.

Tập san Toán học STAR EDUCATION


78 LÊ PHÚC LỮ

Lời giải. Chọn câu C.


Xét bài toán sau: Cho khối nón (N ) có đỉnh A, đáy có tâm là I, bán kính r và chiều
cao h nội tiếp mặt cầu (S) có tâm O, bán kính R. Tìm thể tích lớn nhất của khối
nón này.
Để VN max thì ta xét h ≥ R (vì nếu h < R thì đối xứng đường tròn đáy của (N ) qua
tâm O, ta có bán kính đáy giữ nguyên nhưng chiều cao tăng lên). Khi đó OI = h−R
và r2 = R2 − (h − R)2 = h(2R − h) nên V = 13 πr2 h = 13 π(2R − h)h2 . Theo bất đẳng
thức Cô-si thì  3
h h 2R
(2R − h) · · ≤
2 2 3
8πR3 h 4R
nên V ≤ 81
. Giá trị lớn nhất này đạt được khi 2R − h = 2
⇔h= 3
.
Trở lại bài toán, theo kết quả trên, để V(N ) max thì I ∈ AB sao cho AI = 4R 3
= 2AB3
hay  
−→ 2 −→ 2 8 8 4
AI = AB = (4; 4; 2) = ; ; ,
3 3 3 3 3
−→
trong đó I là tâm đường tròn đáy. Từ đó I 14 ; 11 ; 13 .Ta cũng có AB = (4; 4; 2) k

3 3 3
(2; 2; 1) vuông góc (I) nên mặt phẳng cần tìm có phương trình
14 11 13
2(x − ) + 2(y − ) + (z − ) = 0 ⇔ 2x + 2y + z − 21 = 0.
3 3 3
Vì thế (b, c, d) = (2, 1, −21) nên b + c + d = −18.

Chúc các em học sinh có một mùa thi Đại học thật thành công!

Tập san Toán học STAR EDUCATION

You might also like